INTRODUCTION At one time, the flight engineer functioned as an inflight maintenance person. Today, the flight engineer is a technical expert, who must be thoroughly familiar with the operation and function of various airplane components. The principal function of the flight engineer is to assist the pilots in the operation of the airplane. Specific duties vary with different airplanes and operators. The questions and answers on the flight engineer knowledge tests pertain only to airplanes that require a flight engineer. Because the questions and answers cover a wide scope of airplanes, powerplants, and systems, some questions are general in nature. The information contained in the questions and answers should never take precedence over specific information furnished by a manufacturer in the operation of an airplane. ELIGIBILITY REQUIREMENTS FOR THE KNOWLEDGE TEST The minimum age for taking the knowledge test is 19. A medical certificate is not required. Flight training in the duties of a flight engineer is not required. Applicants must be able to read, speak, and understand the English language. The applicant must hold: 1.an unrestricted commercial pilot certificate with an instrument rating; or 2.an airline transport pilot certificate issued by the FAA or another International Civil Aviation Organization member nation; or 3.an AC Form 8080-2, Airman Written Test Report, or Airman Computer Test Report for a flight engineer original class rating, or a flight engineer certificate, when applying to take an additional class rating test; or 4.an FAA Form 8060-7, Airman's Authorization for Written Test. This form is issued by an FAA inspector upon the applicant’s presenting satisfactory evidence of meeting one of the following practical experience requirements specified by FAR 63.37: a.FAR 63.37(b)(1). The applicant must have at least 3 years of diversified practical experience in aircraft and engine maintenance. At least 1 year of this experience must have been on multiengine aircraft with engines that each have at least 800 horsepower, or the turbine engine equivalent. b.FAR 63.37(b)(2). The applicant must have graduated from a 2-year aircraft and engine maintenance course, of which at least 6 months was devoted to the maintenance of aircraft, with engines that each have at least 800 horsepower, or the turbine engine equivalent. 5 --------------------------------------- 6 c.FAR 63.37(b)(3). The applicant must hold a degree in aeronautical, electrical, or mechanical engineering and 6 months of practical experience in the maintenance of multiengine aircraft, with engines that each have at least 800 horsepower, or the turbine engine equivalent. d.FAR 63.37(b)(5). The applicant must have accumulated 200 hours of flight time as pilot in command, or second in command performing the functions of pilot in command, under the supervision of a pilot in command in an airplane certified in the transport category or a military airplane of equivalent weight and power, with at least two engines. e.FAR 63.37(b)(6). The applicant must have accumulated 200 hours of flight time as a flight engineer in an airplane requiring a flight engineer, or an airplane with at least three engines that each have at least 800 horsepower, or the turbine engine equivalent. f.FAR 63.37(b)(7). Within the previous 90 days, the applicant must have completed the ground portion of an approved Part 63, appendix C, flight engineer training course, for the class rating for which the application has been made. ELIGIBILITY REQUIREMENTS FOR THE ORAL AND FLIGHT TESTS The minimum age for the oral and flight tests is 19. However, to obtain a flight engineer certificate, the minimum age is 21. An applicant who is less than 21 years of age and successfully completes the oral and flight tests will be issued a letter of aeronautical competency. The letter will state that the applicant has met all the requirements for a flight engineer certificate except for age. When an applicant presents proof of reaching age 21, and a second-class medical certificate or better, the letter of aeronautical competency may be exchanged for a temporary airman certificate at any Flight Standards District Office (FSDO). A current second-class medical certificate or better is required for taking the oral and flight tests. Applicants must present a current or validated AC Form 8080-2, Airman Written Test Report, or Airman Computer Test Report. Note: The flight engineer turboprop and basic 300 (FET-300) series tests administered in 1989 and 1990 did not include the flight engineer basic (FEB) test. To be valid, FET-300 series test results must be accompanied by an FEB test result for an original flight engineer certificate. Administration of the flight engineer basic test (FEB) by itself was eliminated in 1991. This change was made to ensure specificity for testing over regulations, theory of flight and aerodynamics, meteorology with respect to engine operations, and center-of-gravity computations. Applicants requiring the basic test to accompany a class test for original certification may do so by taking the appropriate flight engineer combined test. The flight training must be completed in the airplane type which will be used for the tests. The minimum amount of flight training time is 5 hours for applicants qualifying under the provisions of FAR 63.37(b) subparagraphs (1), (2), (3), (4) and (7). Applicants who qualify under the provisions of FAR 63.37(b) subparagraph (7) and hold a commercial pilot certificate or higher with an instrument rating may complete all their flight training in a simulator. There is not a minimum amount of flight training time specified for applicants qualifying under the provisions of FAR 63.37(b) subparagraphs (5) and (6). The applicant must present an authorized instructor's recommendation and verification of the instructor’s eligibility to provide the endorsement, if retesting within 30 days after failing the oral or flight test. For an additional class rating, the applicant must present his or her flight engineer certificate. 6 --------------------------------------- 7 FAR 63.35 REQUIREMENTS FOR THE KNOWLEDGE TEST Applicants must pass a knowledge test on the areas specified by FAR 63.35. These areas are arranged in the following order on the computer administered knowledge tests: FAR's that apply to flight engineer duties. Theory of flight and aerodynamics. Meteorology with respect to engine operations. Operating procedures. 1.Preflight. 2.Normal. 3.Emergency. Airplane equipment. Airplane systems. Limitations. 1.Airplane procedures. 2.Engine operations. Math computations. 1.Engine operations. 2.Fuel consumption. 3.Center of gravity. 4.Airplane loading. DESCRIPTION OF THE KNOWLEDGE TEST All test questions are multiple-choice with three choices of answers. Each question can be answered by the selection of a single response, is independent of other questions, and has equal value. The minimum passing score is 70 percent. Applicants must successfully complete a knowledge test appropriate to the desired rating. The following tests are for original class ratings and each contains 80 questions. Three hours is allowed to take each test. Turbojet and Basic.........................................(FEX) Turboprop and Basic.....................................(FET) Reciprocating and Basic.................................(FEN) Applicants desiring to add a class rating to their flight engineer certificate must successfully complete a knowledge test appropriate to the desired class rating. The following tests are for additional class ratings and each contains 50 questions. Two hours is allowed to take each test. Turbojet........................................................(FEJ) Turboprop.....................................................(FEP) Reciprocating................................................(FER) 7 --------------------------------------- 8 Applicants, who hold a flight engineer basic test and require a class rating test, can take an appropriate combined test for an original class rating (FEX, FET, or FEN), or the appropriate class rating test (FEJ, FEP, or FER). Applicants, who hold a flight engineer class rating test and require the basic test, must take the appropriate combined test for an original class rating (FEX, FET, or FEN). USE OF AIDS, REFERENCE, AND TEST MATERIALS Aids, reference, and test materials within the following guidelines may be used if actual test questions or answers are not revealed. All models of computers, regardless of manufacture, may be used, including hand- held computers designed expressly for aviation use, and also small electronic calculators that perform only arithmetic functions (add, subtract, multiply, and divide). Simple programmable memories, which allow addition to, subtraction from, or retrieval of one number from the memory are permissible. Also, simple functions such as square root or percent keys are permissible. The following guidelines apply: 1.The test administrator will furnish a flight engineer test supplement to answer the questions which reference figures, and two sheets of scratch paper. At the close of the test, the supplement and scratch paper must be returned to the test administrator. In addition, scales, straight edges, protractors, plotters, and electronic or mechanical calculators that are directly related to the test may be used. 2.Permanently inscribed manufacturer's instructions on the front and back of such aids, e.g., formulas, conversions, regulations, signals, weather data, holding pattern diagrams, frequencies, weight and balance formulas, and air traffic control procedures are permissible. 3.The use of electronic calculators is subject to the following limitations: a.Prior to, and upon completion of the test, you must actuate the ON/OFF switch and perform any other function that ensures erasure of any data stored in memory circuits. b.The use of electronic calculators incorporating permanent or continuous-type memory circuits without erasure capability is prohibited. The test administrator may refuse the use of your calculator if the test administrator is unable to determine its use of permanent or continuous-type memory circuits without erasure capability. c.Printouts of data must be surrendered at the completion of the test if the calculator incorporates this design feature. d.The use of magnetic cards, magnetic tapes, modules, computer chips, or any other device upon which prewritten programs or information related to the test can be stored and retrieved, is prohibited. e.The use of any operations booklet or manual of operations containing instructions related to the operation of the calculator, during the test, is not permitted. TAKING A KNOWLEDGE TEST BY COMPUTER The FAA has computer testing centers available nationwide and in other countries. To determine the most convenient testing site, contact the computer testing designees listed in appendix 1. They will provide the hours of operation, costs, and schedule your test date and time. You may cancel your appointment 2 business days before the day the test is scheduled without financial penalty. 8 --------------------------------------- 9 When applying for the test, you must provide positive proof of identification, a permanent mailing address, and documentary evidence of age. The identification must include a current photograph, signature, and actual residential address, if different from your mailing address. This information may be presented in more than one form of identification. Acceptable forms of identification include, but are not limited to, drivers’ licenses, government identification cards, passports, alien residency (green) cards, and military identification cards. Other forms of identification that meet the requirements of this paragraph are acceptable. If you are under 21 years of age, your parent or guardian may accompany you and present an identification of themselves as described previously and attest to your identity. Normally, a test may not be started if the full allotted time is not available before the closing time of the testing center. An exception can be made by signing an agreement statement waiving the right to the full allotted time. When taking the test, keep the following points in mind: 1.Unless you are experienced with taking tests by computer and are familiar with the features, take the sample test. The practice exercises do not count against test time. 2.Read each question carefully before looking at the possible answers. You should clearly understand the question or problem before attempting to solve it. 3.If you are unsure of the answer, mark it for RECALL, and then return to it after answering the other questions on the test. Use caution in selecting an answer different than your first selection. 4.Answer all of the questions. Credit is not given for questions which are not answered. The computer is programmed to alert to any unanswered questions. 5.Do not spend too much time on any one question. Upon completing the test, an airman computer test report is issued with the score and the subject matter codes for any questions which were answered incorrectly. These codes identify the reference materials used in the preparation of the flight engineer test. Review the reference materials to improve your understanding of the subject matter. The total number of test questions missed may differ from the number of subject matter codes shown on the test report, since more than one question for that subject matter may have been missed. RETESTING An applicant who holds an airman computer test report with a passing score may take another knowledge test for that same class rating 30 days after the last test was taken but not before. Eligibility to retake the test is established by presenting the test administrator with the last airman computer test report. The test administrator will retain these test results. The score of the last test taken is the official score. An applicant for a flight engineer certificate who fails a knowledge test may apply for retesting: 1.30 days after the date the test was failed; or 2.after receiving additional instruction that is necessary, in the opinion of the FAA Administrator or the applicant's instructor (if the FAA Administrator has authorized the instructor to determine the additional instruction necessary) to prepare the applicant for retesting. The following persons are authorized to provide the endorsement for a flight engineer applicant to retake the knowledge test within 30 days following failure: 9 --------------------------------------- 10 1.An FAA certificated flight engineer with an appropriate class rating. 2.A U.S. Armed Forces flight engineer instructor, standardization or evaluation flight engineer for that airplane class. 3.An instructor employed by a training facility approved under FAR 63.43. Before retaking a knowledge test, a flight engineer applicant must present to the test administrator the following documentation of the endorser's eligibility: 1.A U.S. Armed Forces flight engineer instructor, standardization, or evaluation flight engineer, for the airplane class being retested, must provide the applicant with copies of flight logs, military qualification forms, or a statement documenting eligibility from his or her military supervisor. 2.An instructor, employed by a training facility approved under FAR 63.43, must provide the applicant with a statement from that facility. The statement must include certification of the endorser's position as an instructor at that facility. The statement on the test report shall be completed by: 1.An FAA certificated flight engineer entering his or her last name and first initial, flight engineer certificate number, class rating, and signature. An example of this would be: Winter, L., FE 2069084, Reciprocating. 2.A U.S. Armed Forces flight engineer instructor, standardization or evaluation flight engineer entering his or her last name and first initial, branch of service, type of designation (instead of an FAA certificate number), and signature. An example of this would be: Winter, K., USN, FE Instructor C-130 (L- 382). 3.An instructor, employed by a training facility approved under FAR 63.43, entering his or her last name and first initial, facility name, position, and signature. An example of this would be: Winter, B., AAL, FE Instructor Turbojet. VALIDITY PERIOD FOR KNOWLEDGE TESTS The flight engineer knowledge test report is valid for 24 calendar months. The validity period may be extended when application is made to take the oral and flight tests, if the following requirements are met: 1.Air Carrier Employees. The following criteria apply to flight crewmembers and mechanics employed by an FAR 121 or FAR 135 air carrier. Employment by an FAR 135 on-demand operator does not qualify an applicant for an extension: a.Applicants who are flight crewmembers must have completed initial new-hire training, initial equipment training, or transition training. b.Applicants who are flight crewmembers must be participating in a training program which includes a recurrent training curriculum in accordance with FAR 121 or FAR 135. c.Applicants who are mechanics must meet the currency requirements of FAR 65. 10 --------------------------------------- 11 d.Applicants must be currently employed by an FAR 121 or an FAR 135 air carrier. However, applicants do not need to have been continuously employed by a qualified air carrier between the time they passed the knowledge test and the time they apply to take the oral and flight tests. 2.Military Applicants. The follo wing criteria apply to military applicants who apply for extensions on the basis of participation in a training program of a scheduled military transport service: a.Applicants must have participated in a flight engineer or maintenance training program at the time of passing the knowledge test or begun a flight engineer or maintenance training program within 24 calendar months after passing the knowledge test. b.Applicants must be currently participating in a military flight engineer or maintenance training program. 3.Continued Eligibility Documentation. Inspectors and examiners will not accept, for the oral test, an expired AC Form 8080-2, Airman Written Test Report, or Airman Computer Test Report, unless the applicant provides written evidence of continued eligibility. When satisfactory evidence is presented, the inspector or examiner will enter, date, and sign the following statement on the test report: “The period of validity of this form has been extended in accordance with the provisions of FAR 63.35(d).” CHEATING OR OTHER UNAUTHORIZED CONDUCT Except as authorized by the FAA, no person may - 1.Copy, or intentionally remove test materials; 2.Give to another, or receive from another, any part or copy of that test; 3.Give help on that test to, or receive help on that test from, any person during the period that test is being given; 4.Take any part of that test in behalf of another person; 5.Use any material or aid during the period that test is being given; or 6.Intentionally cause, assist, or participate in any act prohibited by this paragraph. Any person who commits an act prohibited by this paragraph is not eligible for any airman or ground instructor certificate or rating for a period of 1 year after the date of that act. In addition, the commission of that act is a basis for suspending or revoking any airman or ground instructor certificate or rating held by that person. REPLACING LOST OR DESTROYED TEST REPORTS If an AC Form 8080-2, Airman Written Test Report, or Airman Computer Test Report is lost or destroyed, a duplicate report may by obtained by sending a request to: Federal Aviation Administration Airmen Certification Branch P.O. Box 25082 Oklahoma City, OK 73125 11 --------------------------------------- 12 The request should include your full name, date of birth, social security number, test title, date or approximate date the test was taken, whether it was a written or computer test (if computer test, name of computer testing designee and address), a brief explanation of why the duplicate is being requested, $1 (amount subject to change) money order or check payable to the FAA, return address, daytime telephone number, and signature. If expedited handling is required, it costs approximately $20 more and may be coordinated with the Airman Certification Branch written test department by calling (405) 954-3235. 12 --------------------------------------- 13 SAMPLE TEST QUESTIONS AND ANSWERS 1.What is the air carrier requirement for preflighting the flight engineer's oxygen equipment? AŸThe preflight shall be completed by the flight engineer before each flight. BŸThe preflight may be completed by any flight crewmember before each flight. CŸThe preflight must be completed by the flight engineer for the first flight of the day only. Answer A—Subject Matter Code: D11. FAR 121.337 - Before each flight, each item of protective breathing equipment at flight crewmember duty stations must be checked by the flight crewmember who will use the equipment. 2.The point on an airfoil through which lift acts is the AŸCG. BŸcenter of pressure. CŸmidpoint of the chord. Answer B—Subject Matter Code: T33. The center of pressure is the point at which the chord of an airfoil section intersects the line of action of the resultant aerodynamic forces of lift and drag about which the pressures balance. 3.Which factor has the effect of increasing V1 speed? AŸDry cold air. BŸHigh takeoff gross weight. CŸSlush or standing water on the runway. Answer B—Subject Matter Code: W12. Takeoff performance is affected by gross weight, thrust on the airplane, temperature, pressure altitude, wind direction and velocity, runway slope, and runway surface. Adjustments to V are made for temperature, gross weight, pressure altitude, and flap setting. Some airplane 1 performance tables make a small correction for strong winds. High gross weight, pressure altitude, or temperature will all increase V speed. Slush or water on the runway reduces the stopping performance of 1 the airplane and an aborted takeoff must be started at a lower speed. 4.What does declaring minimum fuel to ATC imply? AŸTraffic priority is needed to the destination airport. BŸEmergency handling is required to the nearest usable airport. CŸAn emergency situation is possible should an undue delay occur. Answer C—Subject Matter Code: J19. Declaring minimum fuel to ATC indicates that upon reaching the destination that an emergency situation is possible should any undue delay occur. The airplane will not receive traffic priority unless an emergency is declared. If the remaining usable fuel supply is such that no delay can be taken, ATC should be notified immediately by declaring an emergency due to low fuel and stating the minutes of fuel remaining. 13 --------------------------------------- 14 5.Which position should be selected on the diluter-demand oxygen regulator if there is smoke in the cockpit? AŸNormal. BŸEmergency. CŸ100 percent. Answer C— Subject Matter Code: S69. Setting the oxygen selector lever to 100 percent closes the outside air passage to the regulator. The outside air passage dilutes the oxygen supplied to the mask with air from the cabin and is open at low altitudes. When the airplane climbs, the passage begins to close until it is completely closed at approximately 34,000 feet. 6.What is residual voltage? AŸVoltage produced that is not in phase with the current. BŸVoltage stored in the generator exciter output windings. CŸVoltage produced by permanent magnets which starts the ac generator output. Answer C— Subject Matter Code: S66. Residual voltage is the voltage of a generator with no field current flowing, and is produced by the residual magnetism of the generator. If the voltmeter indicates residual voltage, the generator is turning. If there is no voltage, the generator has been disconnected, or it has lost its residual magnetism. 7.The purpose of an aileron balance panel is to AŸassist in moving the ailerons. BŸaerodynamically prevent control surface flutter. CŸprovide a balance between the forces in front of the hinge line with moments aft of the hinge line. Answer A—Subject Matter Code: S55. Pressure changes created by the aileron deflect a hinged panel in a compartment ahead of the aileron. Movement of the hinged panel then moves the control surface. The greater the deflection, the greater the pressure changes, and the more assistance will be provided by the hinged panel. 8.Moisture in a pneumatic system may cause AŸcorrosion. BŸa variety of sounds including banging, squealing, and chattering. CŸreturn lines to freeze when the pressure of the air drops during actuation. Answer A—Subject Matter Code: T46. Moisture in a pneumatic system can cause freezing of operating units; interfere with the normal operation of valves, pumps, etc.; and cause corrosion. After the compressed air serves its purpose, it is dumped overboard. 14 --------------------------------------- 15 9.Why should turbine engines normally be operated at idle for a period of time before shutdown? AŸThe turbine case cools faster and may shrink down and seize the turbine blades. BŸRapid cooling of the compressor section may cause cracking of compressor blades. CŸTemperature reduction and stabilization prevents a hot combustion chamber from igniting residual fuel. Answer A—Subject Matter Code: T04. The turbine case and the turbine wheels operate at approximately the same temperature when the engine is running. After shutdown, the turbine case will cool faster than the turbine wheels and may shrink down on the still-rotating turbine wheels if the engine is too hot. Under extreme conditions, the turbine blades may seize. This can be avoided if the engine is cooled at idle speed after prolonged high thrust. 10.Which flight conditions will result in the largest propeller blade angle? AŸInitial climb-out. BŸApproach to landing. CŸHigh-speed, high-altitude cruise flight. Answer C—Subject Matter Code: S18. A constant-speed propeller will attain the largest blade angle when the airplane is at high speed and high altitude. The air is less dense and the propeller requires a larger blade angle for the same amount of torque. 11.If the nosegear retracts forward on an airplane with a datum located forward of the nose, the total moments will AŸincrease. BŸdecrease. CŸremain the same. Answer B—Subject Matter Code: H14. When the landing gear swings forward, the total moments will decrease in proportion to the distance the weight is moved. 12.A cargo airplane is loaded to a maximum takeoff gross weight of 150,000 pounds. How many 150- pound boxes must be moved from Station 1200.0 to Station 700.0 to move the CG forward 3 inches? AŸ3 boxes. BŸ6 boxes. CŸ22 boxes. Answer B—Subject Matter Code: H14. Total weight............................................150,000 lb CG change...........................................................3" Distance weight is shifted.........1200" - 700" = 500" Weight shifted............150,000 x 3" 500" = 900 lb Number of boxes..............900 lb 150 lb = 6 boxes 15 --------------------------------------- 16 APPENDIX 1 16 --------------------------------------- 17 LIST OF REFERENCE MATERIALS AND SUBJECT MATTER KNOWLEDGE CODES The publications listed in the following pages contain study material that may be used in preparing for the flight engineer computer administered knowledge tests. These publications may be purchased through U.S. Government bookstores, or commercial aviation book and supply companies. The latest revision of the references should be requested. The knowledge standards and subject matter knowledge codes for the flight engineer tests are derived from the following reference materials. When reviewing the results of the knowledge test, compare the subject matter knowledge code(s) on the airman test report to these references. FAR 1 Definitions and Abbreviations A01General Definitions A02Abbreviations and Symbols FAR 25Airworthiness Standards: Transport Category Airplanes A03General A04Flight A05Structure A06Design and Construction A07Powerplant A08Equipment A09Operating Limitations and Information FAR 61Certification: Pilots and Flight Instructors A20General FAR 63Certification: Flight Crewmembers Other Than Pilots A30General A31Flight Engineers FAR 91General Operating Rules B07General B14Large and Turbine-Powered Multiengine Airplanes B15Additional Equipment and Operating Requirements for Large and Transport Category Aircraft FAR 121 Certification and Operations: Domestic, Flag and Supplemental Air Carriers and Commercial Operators of Large Aircraft D01General D07Manual Requirements D08Aircraft Requirements D09Airplane Performance Operating Limitations D10Special Airworthiness Requirements D11Instrument and Equipment Requirements 17 --------------------------------------- 18 D12Maintenance, Preventive Maintenance, and Alterations D13Airman and Crewmember Requirements D14Training Program D15Crewmember Qualifications D17Flight Time Limitations and Rest Requirements: Domestic Air Carriers D18Flight Time Limitations: Flag Air Carriers D19Flight Time Limitations: Supplemental Air Carriers and Commercial Operators D20Flight Operations D21Dispatching and Flight Release Rules D22Records and Reports D23Crewmember Certificate: International FAR 125 Certification and Operations: Airplanes Having a Seating Capacity of 20 or More Passengers or a Maximum Payload Capacity of 6,000 Pounds or More D30General D31Certification Rules and Miscellaneous Requirements D32Manual Requirements D33Airplane Requirements D34Special Airworthiness Requirements D35Instrument and Equipment Requirements D36Maintenance D37Airman and Crewmember Requirements D38Flight Crewmember Requirements D39Flight Operations D40Flight Release Rules D41Records and Reports US HMR 175 Materials Transportation Bureau Hazardous Materials Regulations (HMR) G01General Information and Regulations G02Loading, Unloading, and Handling G03Specific Regulation Applicable According to Classification of Material AC 91-23 Pilot's Weight and Balance Handbook H10Weight and Balance Control H11Terms and Definitions H12Empty Weight Center of Gravity H13Index and Graphic Limits H14Change of Weight H16Control of Loading Ÿ Large Aircraft AC 00-6 Aviation Weather I20The Earth's Atmosphere I21Temperature I22Atmospheric Pressure and Altimetry I23Wind I24Moisture, Cloud Formation, and Precipitation I25Stable and Unstable Air 18 --------------------------------------- 19 I26Clouds I27Air Masses and Fronts I28Turbulence I29Icing I30Thunderstorms I31Common IFR Producers I32High Altitude Weather I33Arctic Weather I34Tropical Weather I36Glossary of Weather Terms AIM Airman's Information Manual J03Airport Lighting Aids J04Air Navigation and Obstruction Lighting J05Airport Marking Aids and Signs J11Service Available to Pilots J13Airport Operations J15Preflight J23Distress and Urgency Procedures J25Meteorology J26Altimeter Setting Procedures J27Wake Turbulence J29Potential Flight Hazards J30Safety, Accident, and Hazard Reports J31Fitness for Flight AC 67-2 Medical Handbook for Pilots J52Hypoxia J53Hyperventilation J55The Ears J56Alcohol J57Drugs and Flying J58Carbon Monoxide J59Vision J60Night Flight J61Cockpit Lighting J62Disorientation (Vertigo) J63Motion Sickness J64Fatigue J65Noise J66Age J67Some Psychological Aspects of Flying J68The Flying Passenger ADDITIONAL ADVISORY CIRCULARS K01AC 00-24, Thunderstorms K02AC 00-30, Rules of Thumb for Avoiding or Minimizing Encounters with Clear Air Turbulence K03AC 00-34, Aircraft Ground Handling and Servicing 19 --------------------------------------- 20 K04AC 00-54, Pilot Wind Shear Guide K11AC 20-34, Prevention of Retractable Landing Gear Failure K12AC 20-32, Carbon Monoxide (CO) Contamination in Aircraft Ÿ Detection and Prevention K13AC 20-43, Aircraft Fuel Control K20AC 20-103, Aircraft Engine Crankshaft Failure K40AC 25-4, Inertial Navigation System (INS) L05AC 60-22, Aeronautical Decision Making L15AC 61-107, Operations of Aircraft at Altitudes Above 25,000 Feet MSL and/or MACH numbers (Mmo) Greater Than .75 L34AC 90-48, Pilots' Role in Collision Avoidance L50AC 91-6, Water, Slush, and Snow on the Runway L52AC 91-13, Cold Weather Operation of Aircraft L53AC 91-14, Altimeter Setting Sources L57AC 91-43, Unreliable Airspeed Indications L59AC 91-46, Gyroscopic Instruments Ÿ Good Operating Practices L61AC 91-50, Importance of Transponder Operation and Altitude Reporting L62AC 91-51, Airplane Deice and Anti-Ice Systems L80AC 103-4, Hazard Associated with Sublimation of Solid Carbon Dioxide (Dry Ice) Aboard Aircraft M01 AC 120-12, Private Carriage Versus Common Carriage of Persons or Property M02 AC 120-27, Aircraft Weight and Balance Control M08 AC 120-58, Large Aircraft Ground Deicing M13 AC 121-195-1, Operational Landing Distances for Wet Runways; Transport Category Airplanes M51 AC 20-117, Hazards Following Ground Deicing and Ground Operations in Conditions Conducive to Aircraft Icing M52 AC 00-2, Advisory Circular Checklist AC 65-9A Airframe and Powerplant Mechanics General Handbook S01Mathematics S02Aircraft Drawings S03Aircraft Weight and Balance S04Fuels and Fuel Systems S05Fluid Lines and Fittings S06Aircraft Hardware, Materials, and Processes S07Physics S08Basic Electricity S09Aircraft Generators and Motors S10Inspection Fundamentals S11Ground Handling, Safety, and Support Equipment AC 65-12A Airframe and Powerplant Mechanics Powerplant Handbook S12Theory and Construction of Aircraft Engines S13Induction and Exhaust Systems S14Engine Fuel and Metering Systems S15Engine Ignition and Electrical Systems S16Engine Starting Systems S17Lubrication and Cooling Systems S18Propellers S19Engine Fire Protection Systems S20Engine Maintenance and Operation 20 --------------------------------------- 21 AC 65-15A Airframe and Powerplant Mechanics Airframe Handbook S21Aircraft Structures S22Assembly and Rigging S23Aircraft Structural Repairs S24Ice and Rain Protection S25Hydraulic and Pneumatic Power Systems S26Landing Gear Systems S27Fire Protection Systems S28Aircraft Electrical Systems S29Aircraft Instrument Systems S31Cabin Atmosphere Control Systems 2 EA-ITP-G A and P Technician General Textbook ŸŸ International Aviation Publishers (IAP), Inc., Second Edition S32Mathematics S33Physics S34Basic Electricity S35Electrical Generators and Motors S36Aircraft Drawings S37Weight and Balance S38Fluid Lines and Fittings S39Aircraft Hardware S40Corrosion and Its Control S41Nondestructive Inspection S42Ground Handling and Servicing S43Maintenance Forms and Records S44Maintenance Publications 2 EA-ITP-P A and P Technician Powerplants Textbook ŸŸ IAP, Inc., Second Edition S45Reciprocating Engines S46Turbine Engines S47Engine Removal and Replacement S48Engine Maintenance and Operation S49Induction and Exhaust Systems S50Engine Fuel and Fuel Metering S51Engine Ignition and Electrical Systems S52Engine Lubrication and Cooling Systems S53Engine Fire Protection Systems S54Propellers 2 EA-ITP-A A and P Technician Airframe Textbook ŸŸ IAP, Inc., Second Edition S55Aircraft Structures S56Assembly and Rigging S57Aircraft Fabric Covering S58Aircraft Painting and Finishing S59Aircraft Metal Structural Repair 21 --------------------------------------- 22 S60Aircraft Wood and Composite Structural Repair S61Aircraft Welding S62Ice and Rain Control Systems S63Hydraulic and Pneumatic Power Systems S64Aircraft Landing Gear Systems S65Fire Protection Systems S66Aircraft Electrical Systems S67Aircraft Instrument Systems S68Aircraft Fuel Systems S69Aircraft Cabin Atmosphere Control Systems EA-TEP-2 Aircraft Gas Turbine Powerplants ŸŸ IAP, Inc. S71Jet Propulsion Theory S72Turbine Engine Design and Construction S73Engine Familiarization S74Inspection and Maintenance S75Lubrication Systems S76Fuel Systems S77Compressor Anti-Stall Systems S78Anti-Icing Systems S79Starter Systems S80Ignition Systems S81Engine Instrument Systems S82Fire/Overheat Detection and Extinguishing Systems for Turbine Engines S83Engine Operation The Aircraft Gas Turbine Engine and Its Operation ŸŸ United Technologies Corporation, Pratt & Whitney, 1988 T01Gas Turbine Engine Fundamentals T02Gas Turbine Engine Terms T03Gas Turbine Engine Components T04Gas Turbine Engine Operation T05Operational Characteristics of Jet Engines T06Gas Turbine Engine Performance Aircraft Powerplants ŸŸ Glencoe/McGraw-Hill, Seventh Edition T07Aircraft Powerplant Classification and Progress T08Reciprocating-Engine Construction and Nomenclature T09Internal-Combustion Engine Theory and Performance T10Lubricants and Lubricating Systems T11Induction Systems, Superchargers, Turbochargers, and Cooling and Exhaust Systems T12Basic Fuel Systems and Carburetors T13Fuel Injection Systems T14Reciprocating-Engine Ignition and Starting Systems T15Operation, Inspection, Maintenance, and Troubleshooting of Reciprocating Engines T16Reciprocating-Engine Overhaul Practices T17Gas Turbine Engine: Theory, Construction, and Nomenclature T18Gas Turbine Engine: Fuels and Fuel Systems 22 --------------------------------------- 23 T19Turbine-Engine Lubricants and Lubricating Systems T20Ignition and Starting Systems of Gas-Turbine Engines T21 Turbofan Engines T22Turboprop Engines T23Turboshaft Engines T24Gas-Turbine Operation, Inspection, Troubleshooting, Maintenance, and Overhaul T25Propeller Theory, Nomenclature, and Operation T26Turbopropellers and Control Systems T27Propeller Installation, Inspection, and Maintenance T29Engine Indicating, Warning, and Control Systems EA-ATD-2 Aircraft Technical Dictionary ŸŸ IAP, Inc. T30Definitions Aircraft Basic Science ŸŸ Glencoe/McGraw-Hill, Seventh Edition T31Fundamentals of Mathematics T32Science Fundamentals T33Basic Aerodynamics T34Airfoils and their Applications T35Aircraft in Flight T36Aircraft Drawings T37Weight and Balance T38Aircraft Materials T39Fabrication Techniques and Processes T40Standard Aircraft Hardware T41Aircraft Fluid Lines and their Fittings T43Ground Handling and Safety T44Aircraft Inspection and Servicing Aircraft Maintenance and Repair ŸŸ Glencoe/McGraw-Hill, Sixth Edition T45Aircraft Structures T46Aircraft Fluid Power Systems T47Aircraft Landing-Gear Systems T48Aircraft Fuel Systems T49Environmental Systems T50Aircraft Instruments and Instrument Systems T51Auxiliary Systems T52Assembly and Rigging EA-363 Transport Category Aircraft Systems ŸŸ IAP, Inc. T53Types, Design Features and Configurations of Transport Aircraft T54Auxiliary Power Units, Pneumatic, and Environmental Control Systems T55Anti-Icing Systems and Rain Protection T56Electrical Power Systems T57Flight Control Systems T58Fuel Systems T59Hydraulic Systems 23 --------------------------------------- 24 T60Oxygen Systems T61Warning and Fire Protection Systems T62Communications, Instruments, and Navigational Systems T63Miscellaneous Aircraft Systems and Maintenance Information Aircraft Electricity and Electronics ŸŸ Glencoe/McGraw-Hill, Fifth Edition T64Fundamentals of Electricity T65Applications of Ohm's Law T66Aircraft Storage Batteries T67Alternating Current T68Electrical Wire and Wiring Practices T69Electrical Control Devices T70Electric Measuring Instruments T71Generators and Related Control Circuits T72Alternators, Inverters, and Related Controls T73Electric Motors T74Power Distribution Systems T75Design and Maintenance of Aircraft Electrical Systems EA-338 Flight Theory for Pilots ŸŸ IAP, Inc., Third Edition W01 Introduction W02 Air Flow and Airspeed Measurement W03 Aerodynamic Forces on Airfoils W04 Lift and Stall W05 Drag W06 Jet Aircraft Basic Performance W07 Jet Aircraft Applied Performance W08 Prop Aircraft Basic Performance W09 Prop Aircraft Applied Performance W11 Hazards of Low Speed Flight W12 Takeoff Performance W13 Landing Performance W14 Maneuvering Performance W15 Longitudinal Stability and Control W16 Directional and Lateral Stability and Control W17 High Speed Flight Fly the Wing, ŸŸ Iowa State University Press/Ames, Second Edition X01Basic Aerodynamics X02High-Speed Aerodynamics X03High-Altitude Machs X07Takeoffs X08Rejected Takeoffs X09Climb, Cruise, and Descent X20Weight and Balance X21Flight Planning X22Icing X23Use of Anti-ice and Deice 24 --------------------------------------- 25 X24Winter Operation X25Thunderstorm Flight X26Low-Level Wind Shear Aircraft Gas Turbine Engine Technology,ŸŸGlencoe/McGraw-Hill, Second Edition Y01History and Theory Y02Construction and Design Y03Systems and Accessories Y04Maintenance and Testing Y05Representative Engines Y06Appendixes NOTE: AC 00-2, Advisory Circular Checklist, transmits the current status of all FAA advisory circulars (AC’s), as well as FAA internal publications and miscellaneous flight information such as Airman’s Information Manual (AIM), Airport/Facility Directory, knowledge test guides, practical test standards, and other material directly related to a certificate or rating. To obtain a free copy of AC 00-2, send your request to: U.S. Department of Transportation General Services Section, M-45.3 Washington, DC 20590 25 --------------------------------------- 26 COMPUTER TESTING DESIGNEES The following is a list of the computer testing designees authorized to give FAA knowledge tests. This list should be helpful in choosing where to register for a test or for requesting additional information. Aviation Business Services 1-800-947-4228 outside U.S. (415) 259-8550 Drake Prometric 1-800-359-3278 outside U.S. (612) 896-7702 Sylvan Learning Systems, Inc. 1-800-967-1100 outside U.S. (410) 880-0880, Extension 8890 The latest listing of computer testing center locations may be obtained through FedWorld, (703) 321-3339, in the FAA library file named TST_SITE. For technical assistance, contact the FedWorld help desk at (703) 487- 4608. 26 ------------------------ --------------------------------------- 1 The following sample questions for flight engineer tests are suitable study questions for the Flight Engineer Turbojet Airman Knowledge Test (FEX). The full FEX test is 80 questions. Please note that the FEX and Flight Engineer Turbojet Added Rating (FEJ) tests share many questions. Students for the FEX or FEJ would do well to study both sets of questions. The FAA computer-assisted testing system is supported by a series of supplement publications. These publications, available through several aviation publishers, include the graphics, legends, and maps that are needed to successfully respond to certain test items. Use the following URL to download a complete list of associated supplement books: http://www.faa.gov/training_testing/testing/airmen/test_questions/ The Learning Statement Reference Guide for Airman Knowledge Testing contains listings of learning statements with their associated codes. It can be located at: http://www.faa.gov/training_testing/testing/airmen/media/LearningStatementReferenceGuide.pdf SAMPLE FEX EXAM: PLT315 1. The speed at which the airflow over the wing first reaches the speed of sound is known as the A. Reynolds number. B. transonic index. C. critical Mach number. PLT315 2. Mach number is commonly defined as the A. ratio of true airspeed to the speed of sound. B. ratio of equivalent airspeed to the speed of sound. C. speed of sound under conditions of standard pressure and temperature. PLT242 3. Compared to a no-wind condition, what effect would a 20 knot headwind component have on takeoff performance? A. The effect of wind on initial acceleration will result in a longer takeoff roll. B. The airplane will reach critical engine failure indicated airspeed at a lower groundspeed. C. Critical engine failure speed and actual groundspeed will be the same as in a zero-wind condition. PLT244 4. When will power applications cause the greatest change in airplane trim and stability? A. When on a power approach at low airspeeds. B. Operation at high gross weight and low airspeed. C. When power is applied simultaneously with a configuration change. PLT214 5. Shock-induced separation of airflow occurring symmetrically near the wing root of a sweptback wing may result in A. severe porpoising due to an attempt to recover control while under reverse command. B. a high-speed stall and sudden pitchup due to the center of pressure moving forward on the wing. C. a severe diving moment, due to the center of pressure moving aft on the wing and a decrease of downwash on the horizontal tail. PLT237 6. What effect will decreasing air density have on lift and drag? --------------------------------------- 2 A. Lift and drag will decrease. B. Lift will increase and drag will decrease. C. Lift will decrease and drag will increase. PLT303 7. The angle of attack which produces the highest L/D ratio A. increases as weight or altitude is increased. B. remains constant regardless of weight or altitude. C. remains constant as altitude is changed, but decreases as weight is reduced. PLT480 8. An airplane is in equilibrium when A. there are no accelerations and the airplane continues in steady flight. B. the airplane is disturbed from its flightpath and it will return without control use. C. the airplane has neither the tendency to continue or return from disturbance displacement. PLT168 9. During flight with zero angle of attack, the pressure along the upper surface of the wing will be A. equal to atmospheric pressure. B. less than atmospheric pressure. C. greater than the pressure below the wing. PLT244 10. The purpose of airplane wing dihedral angle is to A. increase lateral stability. B. increase longitudinal stability. C. increase lift coefficient of the wing. PLT244 11. What is the primary source of directional stability for an airplane? A. CG position. B. Vertical tail. C. Horizontal tail. PLT124 12. An airplane is climbing at Mach .78. The true airspeed will A. increase with altitude. B. increase as pressure decreases. C. decrease as the temperature decreases. PLT011 13. (Refer to figure 2.) Compute the V speeds for the following conditions. Gross weight 250,000 lb Pressure altitude 428 ft OAT +80 °F Flaps 25° Tailwind 5 kts Airport SEA RWY 34 A. V1 118, VR 132, V2 145. B. V1 117, VR 133, V2 144. C. V1 121, VR 133, V2 144. PLT135 14. To which elevation should the cabin altitude be set for the following landing conditions? Altimeter 30.12 Field elev 6172 ft --------------------------------------- 3 Airplane cabin depressurized 500 ft AGL Cabin pressure controller calibrated to 29.92 A. 6,472 feet. B. 6,672 feet. C. 6,792 feet. PLT188 15. (Refer to figure 6.) Which is the lowest cabin altitude that can be maintained at FL 320 with a pressure differential of 14.2 inches Hg? A. -1,000 feet. B. +1,000 feet. C. +8,000 feet. PLT135 16. To which elevation should the cabin altitude be set for the following landing conditions? Altimeter 30.12 Field elev 1295 ft Airplane cabin pressurized to 200 ft below field elev Cabin barometric pressure reference setting 29.92 A. 895 feet. B. 1,295 feet. C. 1,595 feet. PLT007 17. (Refer to figure 13.) Determine the go-around EPR's for these conditions. Pressure altitude 1,000 ft TAT 0 °C A/C bleeds No. 2 and 3 ON No. 1 OFF Anti-ice Eng. ON A. Eng. 1, 2.12; Eng. 2, 2.15; Eng. 3, 2.12. B. Eng. 1, 2.16; Eng. 2, 2.11; Eng. 3, 2.16. C. Eng. 1, 2.16; Eng. 2, 2.08; Eng. 3, 2.12. PLT011 18. (Refer to figures 14 and 15.) Determine the maximum takeoff power settings. Pressure altitude Sea Level OAT +15 °C A/C bleed No. 1 and 2 OFF No. 3 ON Eng. anti-ice OFF No. 2 Eng. EPR gauge Inoperative A. Eng. 1, 2.12; Eng. 2, 98.2; Eng. 3, 2.10. B. Eng. 1, 2.10; Eng. 2, 92.7; Eng. 3, 2.14. C. Eng. 1, 2.14; Eng. 2, 96.9; Eng. 3, 2.10. PLT011 19. (Refer to figure 6.) The maximum temperature limitation for takeoff is ISA +34 °C. Which is the highest temperature that will allow a takeoff from a 7,000-foot pressure altitude airport? A. +87 °F. B. +91 °F. C. +94 °F. PLT016 20. How many minutes of dump time would be required to reach maximum landing weight at touchdown under the following conditions? --------------------------------------- 4 Number of engines 3 Cruise weight 171,000 lb Max. landing weight 142,500 lb Average fuel flow during dumping and descent to touchdown 3,170 lb/hr/eng Time from start dump to landing 19 min Fuel dump rate 2,300 lb/min A. 7.7 minutes. B. 11.1 minutes. C. 12.4 minutes. PLT012 21. (Refer to figure 26.) How much fuel remains after dumping under operating conditions No. 1? A. 4,540 pounds. B. 4,980 pounds. C. 5,100 pounds. PLT012 22. An airplane has been cruising for 2 hours and 15 minutes at a speed of Mach .82. Total fuel consumed during this period has been 27,250 pounds. If Mach 1.0 is 595 knots, what has been the NM per 1,000 pounds of fuel? A. 40.3 NM/1,000 pounds. B. 43.7 NM/1,000 pounds. C. 46.4 NM/1,000 pounds. PLT117 23. Which is a reason for heating cockpit windows? A. Deicing. B. Anti-icing. C. Prevent thermal shock. PLT263 24. A turbojet aircraft is equipped with heated inlet ducts and airfoil leading edges. When is this type of anti-icing system usually activated during flight? A. It is operated continuously while in flight. B. At all times when the OAT is below freezing. C. Whenever icing conditions are first encountered or are expected to occur. PLT108 25. In an electrically heated windshield system, what maintains normal windshield temperature? A. Thermistors. B. Electronic amplifiers. C. Thermal overheat switches. PLT108 26. What will cause the EPR indication to decrease falsely if the Pt probe at the engine nose dome is iced closed? A. The Pt probe vent will act as a Ps probe. B. Anti-ice air will pressurize the nose dome and the vent hole causing a decrease in EPR. C. The reduced inlet area causes pressure to increase, and magnifies the influence of ram air pressure. PLT109 27. Why is it necessary to periodically completely discharge and recharge a nickel-cadmium battery? A. To restore electrolyte levels. --------------------------------------- 5 B. To eliminate cell imbalance and loss of capacity. C. To dissolve nickel oxide formations on positive cells to restore capacity. PLT207 28. What is the function of the circuit breaker in the instrument lighting system? A. Protects the lights from too much current. B. Protects the wiring from too much current. C. Prevents excessive voltage from reaching the wiring. PLT207 29. Aircraft fuse capacities are rated in A. volts. B. watts. C. amperes. PLT207 30. What is an advantage of using 115 volts, 400-Hz alternating current? A. High voltage and low current reduces wire size and weight requirements. B. Commutators may be used with ac motors to decrease repair intervals and costs. C. The ac single-phase induction motors are self-starting, making it possible to use lightweight motors. PLT326 31. Why must oxygen not be permitted to come in contact with oil, grease, or solvents? A. Oxygen is highly flammable and will cause petroleum products to burn or explode. B. Oxygen contact with petroleum products can cause spontaneous fires or explosions. C. Gaseous oxygen is chemically unstable and will combine with petroleum to form a highly explosive mixture. PLT326 32. What is an advantage of a chemical over a gaseous oxygen system? A. Fire hazards are reduced by eliminating oxygen lines. B. Chemical systems may be shutoff at any time after they are activated. C. Reliability is improved by interconnecting individual chemical units. PLT326 33. What type of oxygen system is used for passengers? A. Demand. B. Constant-flow. C. Diluter-demand. PLT326 34. Which position should be selected on the diluter-demand oxygen regulator if there is smoke in the cockpit? A. Normal. B. Emergency. C. 100 percent. PLT137 35. Which components make up the basic air-cycle cooling system? A. Heaters, coolers, and compressor. B. Ram air source, compressors, and engine bleeds. C. A source of compressed air, heat exchangers, and a turbine. PLT366 36. The purpose of a control tab is to --------------------------------------- 6 A. move the flight controls in the event of manual reversion. B. reduce control forces by deflecting in the proper direction to move a primary flight control. C. prevent a control surface from moving to a full deflection position due to aerodynamic forces. PLT346 37. Which direction from the primary control surface does an antiservo tab move? A. Same direction. B. Opposite direction. C. Remains fixed for all positions. PLT473 38. A purpose of ground spoilers is to A. reduce the wing's lift upon landing. B. aid in rolling an airplane into a turn. C. increase the rate of descent without gaining airspeed. PLT499 39. Total air temperature is equal to A. OAT corrected for altitude. B. ambient temperature minus the ram rise from adiabatic compression of the boundary layer. C. ram air temperature when the recovery factor of the temperature sensor is equal to 100 percent. PLT342 40. Which is a means of controlling the fuel temperature on turbojet-powered airplanes? A. Electrically heated fuel filters. B. Engine bleed air routed to a heat exchanger. C. Fuel filters heated by engine lubricating oil. PLT342 41. Oil extracts the most heat from which turbine engine components? A. Turbine bearings. B. Compressor bearings. C. Accessory drive bearings. PLT251 42. What is the difference between Jet A and Jet B fuel? A. Jet A is colorless and Jet B is straw colored. B. Jet A has a freeze point of -40 °C and Jet B has a freeze point of -47 °C. C. Jet B is a blend of gasoline and kerosene made for operating at extremely low temperatures. PLT273 43. One purpose of a hydraulic accumulator is to A. compress hydraulic fluid. B. absorb sudden pressure surges. C. store hydraulic fluid from small system leaks. PLT273 44. What is the significance of the pop-out indicator on some hydraulic filters? A. Confirmation of fluid thermal expansion in the gear, flaps, or other systems. B. Evidence that contaminants may prevent components such as check valves from operating. C. Indication of overfilling and the fluid level of the hydraulic reservoir should be checked. PLT273 45. What should be used to remove Skydrol from your skin? A. Solvent. --------------------------------------- 7 B. Soap and water. C. Trichlorethylene. PLT273 46. What should hydraulic flexible hose be inspected for on preflight? A. Slack between fittings. B. Hose supports at least every eighteen inches. C. Layline identification marks spiral clockwise. PLT273 47. Why should synthetic hydraulic fluid be stored in an airtight container? A. High evaporation rate. B. Vapor is extremely toxic. C. Atmospheric moisture contamination. PLT110 48. What item of the antiskid system enables full braking action during taxi and parking? A. The locked-wheel arming circuit is inoperative at taxi speeds. B. The antiskid switch is manually disarmed on the afterlanding checklist. C. The wheel-speed sensors automatically disarm the antiskid system at speeds below about 20 miles per hour. PLT138 49. The purpose of chines on tires is to A. increase traction on snow or ice covered runways. B. reduce the tendency to hydroplane on wet runways. C. deflect water or slush away from the engine intakes. PLT138 50. The purpose of fusible plugs in aircraft wheels is to A. prevent tire blowouts. B. quickly deflate tires for repair. C. protect the antiskid electrical system. PLT139 51. When will the landing gear position warning system provide a warning in the cockpit? A. When the power levers are retarded and the gear is in transit. B. When the power levers are advanced and the speed brakes are not retracted. C. When one power lever is retarded below cruise and the gear is down and locked. PLT337 52. During the walkaround inspection, you observe covers over the pitot probes. Which items will be affected if the covers are not removed? A. Airspeed, altimeter, and autopilot. B. Flight recorder, airspeed, and autopilot. C. Flight recorder, autopilot, instantaneous vertical speed indicator, and airspeed. PLT041 53. What will result if the instrument static pressure line becomes disconnected inside a pressurized cabin during cruise flight? A. The altimeter and airspeed indicator will both read low. B. The altimeter and airspeed indicator will both read high. C. The altimeter will read low and the airspeed indicator will read high. PLT342 --------------------------------------- 8 54. What condition will cause a high EGT, Wf, and RPM with a normal EPR indication, at all power settings? A. FOD. B. Bleed valve stuck open. C. Ice in the inlet pressure lines. PLT477 55. What recovery would be appropriate in the event of compressor stall? A. Reduce the thrust lever and then rapidly advance the thrust lever to decrease the angle of attack on the compressor blades, creating more airflow. B. Reduce the thrust lever and then slowly advance the thrust lever again to decrease the angle of attack on one or more compressor blades. C. Advance the thrust lever slowly to increase airflow and decrease the angle of attack on one or more compressor blades. PLT499 56. Which engine instrument is most likely to show an elevated reading if the turbine wheel is damaged? A. N2. B. EPR. C. EGT or TIT. PLT479 57. It is important to monitor the EGT when starting the engines to prevent A. hot section burnout or metal distortion. B. compressor temperature limits from being exceeded. C. dangerous gas temperatures and velocities when accelerating to idle from injuring personnel or damaging the engine. PLT479 58. Which action should be completed immediately during the start sequence if the ITT attempts to exceed the temperature limit? Shut off the A. fuel. B. starter. C. ignition. PLT499 59. The purpose of the diffuser section in a turbine engine is to A. convert pressure to velocity. B. increase pressure and reduce velocity. C. reduce pressure and increase velocity. PLT499 60. (Refer to figure 7.) The internal engine pressure will be the lowest at which location? A. Location 4. B. Location 5. C. Location 6. PLT479 61. If a turbine engine catches fire internally during the start cycle, what initial action should be taken? A. Secure all switches and leave the airplane. B. Shut off the fuel and continue motoring the starter. C. Discharge CO2 into the inlet duct while continuing to motor the engine. PLT108 --------------------------------------- 9 62. The purpose of diluting ethylene glycol deicing fluid with water in nonprecipitation conditions is to A. raise the eutectic point. B. decrease the freeze point. C. increase the minimum freeze point (onset of crystallization). PLT108 63. Deicing fluid should be dispensed at what temperature? A. Cold. B. Heated. C. Ambient. PLT108 64. Anti-icing fluid should provide freezing point protection to A. -20 °F ambient temperature. B. +32 °F outside temperature or below. C. a freezing point no greater than 20 °F below the ambient or airplane surface temperature. PLT108 65. Type 2 deicing/anti-icing fluids have a significantly A. longer holdover period than type 1 fluids. B. shorter holdover period than type 1 fluids. C. lower viscosity than type 1 fluids. PLT208 66. Which frequency is preferred to declare an emergency to ATC? A. 121.5 Mhz VHF. B. 243.0 Mhz UHF. C. The one in use. PLT497 67. Which transponder code means the airplane is being forced to a new destination? A. 7500. B. 7600. C. 7700. PLT212 68. What identifies a fire extinguisher used for brake fires? A. A square with the letter B. B. A circle with the letter C. C. A star with the letter D. PLT440 69. Which flight crewmember nonessential conversation is allowed below 10,000 feet? A. Discussing stock market reports during taxi. B. None. C. Confirming airplane logbook entries during climb when clear of the airport traffic area. PLT409 70. What is the flight time limitation for flag operations that requires two pilots and at least one additional flight crewmember? A. 100 hours during any 30-day period. B. 300 hours during any 3 calendar months. C. 1,000 hours during any 12 calendar-month period. PLT451 --------------------------------------- 10 71. Which requirement must be met by all flight engineers every 6 months before they can serve on an air carrier flight under 14 CFR part 121? A. Line check or route check. B. Recurrent flight and ground training. C. 50 hours of flight time or a flight check. PLT409 72. Duty and rest period rules for domestic air carrier operations require that a flight crewmember A. not be assigned to any duty with the air carrier during a required rest period. B. be relieved of all duty for at least 48 hours during any 7 consecutive days. C. not be assigned to any duty for a period of at least 18 hours if the flight crewmember had been on duty aloft for 9 hours. PLT438 73. How much supplemental oxygen must pressurized air carrier transport airplanes carry for each flight crewmember on flight deck duty when operating at flight altitudes above 10,000 feet? A. A minimum of 1 hours' supply. B. A minimum of 2 hours' supply. C. A minimum of 30 minutes' supply. PLT439 74. Which maintenance task may a flight engineer perform while operating under 14 CFR part 125? A. Landing light replacement if there is no certificated mechanic available. B. Remove, inspect, and replace a chip detector if the malfunction occurs in a remote area. C. Replenish hydraulic fluid in accordance with applicable regulations and the certificate holder’s manuals. PLT386 75. If a person has lost their flight engineer certificate, the privileges of that certificate may be exercised until a duplicate is received, after obtaining a A. confirming telegram from the FAA which is valid for 30 days. B. FAX from the Airman Certification Branch in Oklahoma City which is valid for 60 days. C. temporary certificate issued by a Flight Standards District Office which is valid for 90 days. PLT203 76. Which is true concerning the troposphere? A. It extends to a uniform height at all latitudes. B. It is thicker over the Equator than over the poles. C. It is the dividing line between the stratosphere and the atmosphere. PLT274 77. An in-flight condition necessary for structural icing to form is A. visible water such as supercooled rain or cloud droplets. B. aerodynamic cooling of an airfoil to 0 degrees C. C. the temperature at the point where moisture strikes the aircraft must be 0 degrees F or colder. PLT021 78. (Refer to figure 52.) What is the maximum payload under operating conditions No. 1? A. 20,500 pounds. B. 21,500 pounds. C. 25,500 pounds. PLT021 79. (Refer to figure 34.) What is the new CG after adding weight under operating conditions No. 1? --------------------------------------- 11 A. 20.9 percent. B. 25.8 percent. C. 27.9 percent. PLT003 80. Before a cargo change is made, the following information is known about an airplane. Aircraft weight 175,000 lb CG 29.5 percent of MAC Length of MAC 860.2 to 1040.9 in If 6,500 pounds of cargo is removed from an average location of Station 1170.0, what is the new CG relative to MAC? A. 24.0 percent. B. 26.8 percent. C. 27.6 percent. ---------------- 08/25/2011 Bank: (Flight Engineer) Airman Knowledge Test Question Bank The FAA computer-assisted testing system is supported by a series of supplement publications. These publications, available through several aviation publishers, include the graphics, legends, and maps that are needed to successfully respond to certain test items. Use the following URL to download a complete list of associated supplement books: http://www.faa.gov/training_testing/testing/airmen/test_questions/ The Learning Statement Reference Guide for Airman Knowledge Testing contains listings of learning statements with their associated codes. It can be located at: http://www.faa.gov/training_testing/testing/airmen/media/LearningStatementReferenceGuide.pdf 1 PLT305 FEX A purpose of leading edge slats on high performance wings is to A) increase lift at relative slow speeds. B) improve aileron control during low angles of attack. C) direct air from the low-pressure area under the leading edge along the top of the wing. 2 PLT305 FEX If the leading edge slats (flaps) will not extend, you A) know lift will be less at slower speeds. B) expect aileron control to improve during low angles of attack. C) anticipate increased air flow over the trailing edge flaps and a nose lifting force result. 3 PLT315 FEX The speed at which the airflow over the wing first reaches the speed of sound is known as the A) Reynolds number. B) transonic index. C) critical Mach number. 4 PLT134 FEX Compared to a no-wind condition, what effect would a 20 knot headwind component have on takeoff performance? A) The effect of wind on initial acceleration will result in a longer takeoff roll. B) The airplane will reach critical engine failure indicated airspeed at a lower groundspeed. C) Critical engine failure speed and actual groundspeed will be the same as in a zero-wind condition. 5 PLT244 FEX When will power applications cause the greatest change in airplane trim and stability? A) When on a power approach at low airspeeds. B) Operation at high gross weight and low airspeed. C) When power is applied simultaneously with a configuration change. 6 PLT214 FEX Shock-induced separation of airflow occurring symmetrically near the wing root of a sweptback wing may result in A) severe porpoising due to an attempt to recover control while under reverse command. B) a high-speed stall and sudden pitchup due to the center of pressure moving forward on the wing. C) a severe diving moment, due to the center of pressure moving aft on the wing and a decrease of downwash on the horizontal tail. 7 PLT214 FEX The purpose of sweeping wings back approximately 30° to 35° is to A) minimize dutch roll. B) reduce high-speed drag. C) provide aileron control when the root of the wing approaches the critical angle of attack. 8 PLT245 FEX The true airspeed at which an airplane stalls varies with A) load factor and angle of attack. B) load factor, weight, and density altitude. C) density altitude, weight, and angle of attack. 9 PLT237 FEX What effect will decreasing air density have on lift and drag? A) Lift and drag will decrease. B) Lift will increase and drag will decrease. C) Lift will decrease and drag will increase. 10 PLT242 FEX An airplane is in equilibrium when A) there are no accelerations and the airplane continues in steady flight. B) the airplane is disturbed from its flightpath and it will return without control use. C) the airplane has neither the tendency to continue or return from disturbance displacement. 11 PLT095 FEX During flight with zero angle of attack, the pressure along the upper surface of the wing will be A) equal to atmospheric pressure. B) less than atmospheric pressure. C) greater than the pressure below the wing. 12 PLT244 FEX The purpose of airplane wing dihedral angle is to A) increase lateral stability. B) increase longitudinal stability. C) increase lift coefficient of the wing. 13 PLT124 FEX An airplane is climbing at Mach .78. The true airspeed will A) increase with altitude. B) increase as pressure decreases. C) decrease as the temperature decreases. 14 PLT124 FEX What must happen to true airspeed to maintain the same angle of attack in level flight, when the air density changes? A) The airspeed must increase when the air density decreases. B) The airspeed must increase when the air density increases. C) The airspeed must decrease when the air density decreases. 15 PLT124 FEX When the air density changes, how must the true airspeed be changed to maintain the same angle of attack in level flight? A) The airspeed must increase when the air density decreases. B) The airspeed must increase when the air density increases. C) The airspeed must decrease when the air density decreases. 16 PLT124 FEX Which is an advantage of flying a jet at high altitudes? A) Lower temperatures increase engine efficiency. B) Thrust increases as the density of the air decreases. C) Lower engine RPM's will result in decreased specific fuel consumption. 17 PLT328 FEX An airplane is flying at a constant altitude with a power setting which produces the maximum air miles per pound of fuel. To maintain the maximum air miles per pound of fuel as the weight of the fuel decreases the engine power setting should be A) decreased. B) increased. C) maintained. 18 PLT328 FEX Which factor is most significant when determining the optimum cruise altitude available? A) Winds aloft and temperature forecast. B) Fuel requirement to climb to altitude. C) Gross weight of the airplane at the beginning of the cruise. 19 PLT002 FEX (Refer to figure 2.) Compute the V speeds for the following conditions. A) V1 118, VR 132, V2 145. B) V1 117, VR 133, V2 144. C) V1 121, VR 133, V2 144. 20 PLT123 FEX Takeoff speed limits V1, VR, and V2, contained in performance charts and tables of the airplane flight manual are A) true airspeeds. B) indicated airspeeds. C) corrected airspeeds. 21 PLT135 FEX (Refer to figure 6.) The cabin pressure altitude is 6,000 feet and the airplane altitude is FL 180. What is the differential pressure? A) 4.44 PSI. B) 4.71 PSI. C) 5.46 PSI. 22 PLT019 FEX (Refer to figure 6.) The cabin pressure differential is 5.46 PSI and the airplane altitude is FL 200. What is the cabin altitude? A) 3,200 feet. B) 4,400 feet. C) 5,000 feet. 23 PLT019 FEX To which elevation should the cabin altitude be set for the following landing conditions? A) 6,472 feet. B) 6,672 feet. C) 6,792 feet. 24 PLT220 FEX Which is a characteristic of the constant Mach cruise control procedure? A) EPR is increased as aircraft weight decreases. B) Thrust is reduced as aircraft weight decreases. C) True airspeed decreases as the outside air temperature (OAT) increases. 25 PLT007 FEX (Refer to figure 13.) Determine the go-around EPR's for these conditions. A) Eng. 1, 2.12; Eng. 2, 2.15; Eng. 3, 2.12. B) Eng. 1, 2.16; Eng. 2, 2.11; Eng. 3, 2.16. C) Eng. 1, 2.16; Eng. 2, 2.08; Eng. 3, 2.12. 26 PLT130 FEX The ratio of NM per hour to fuel flow in pounds per hour identifies which item relating to airplane performance? A) Specific range. B) Specific fuel flow. C) Specific fuel consumption. 27 PLT078 FEX All 14 CFR 139 airports must report A) accident and incident data annually. B) noise complaint statistics for each departure procedure or runway. C) declared distances for each runway. 28 PLT011 FEX (Refer to figures 20 and 21.) What is the takeoff power available under operating conditions No. 1? A) 3,710 shaft horsepower. B) 3,770 shaft horsepower. C) 4,000 shaft horsepower. 29 PLT011 FEX (Refer to figures 22 and 23.) What is the takeoff power setting under operating conditions No. 1? A) 234 BMEP. B) 204 BMEP. C) 59.5 inches manifold pressure. 30 PLT012 FEX (Refer to figure 27.) What is the total estimated fuel burn under operating conditions No. 1? A) 12,800 pounds. B) 14,140 pounds. C) 22,160 pounds. 31 PLT012 FEX (Refer to figure 29.) How much fuel remains after operating under these conditions? A) 3,872 gallons. B) 4,194 gallons. C) 4,309 gallons. 32 PLT016 FEX The captain says to load on 10,000 pounds of fuel. The fuel is 6.5 pounds per gallon. How many liters should the flight engineer order? A) 5,825.00 B) 6,500.00 C) 7,110.00 33 PLT016 FEX (Refer to figure 26.) How much fuel remains after dumping under operating conditions No. 1? A) 4,540 pounds. B) 4,980 pounds. C) 5,100 pounds. 34 PLT012 FEX An airplane has been cruising for 2 hours and 15 minutes at a speed of Mach .82. Total fuel consumed during this period has been 27,250 pounds. If Mach 1.0 is 595 knots, what has been the NM per 1,000 pounds of fuel? A) 40.3 NM/1,000 pounds. B) 43.7 NM/1,000 pounds. C) 46.4 NM/1,000 pounds. 35 PLT127 FEX For an airplane with a given gross weight and constant cruise speed, what is the relationship between fuel flow, temperature, and altitude? Fuel flow is higher when A) both temperature and altitude are decreased. B) both temperature and altitude are increased. C) temperature is increased and altitude is decreased. 36 PLT274 FEX Ice accumulation is usually negligible at temperatures below A) 0 °C. B) -20 °C. C) -40 °F. 37 PLT344 FEX What is the lowest temperature that water droplets may remain in a liquid state? A) 0 °C. B) 0 °F. C) -40 °C. 38 PLT263 FEX A turbojet aircraft is equipped with bleed air heated inlet ducts and airfoil leading edges. When this type of anti-icing system is activated during flight, the flight engineer A) can ignore the turbine temperatures due to the extremely cold outside temperature. B) may need to reduce the power setting to maintain the turbine temperatures (EGT, ITT, etc.) C) will expect to see a decrease in turbine temperatures due to limited combustion air in the engines. 39 PLT263 FEX A turbojet aircraft is equipped with heated inlet ducts and airfoil leading edges. When is this type of anti-icing system usually activated during flight? A) It is operated continuously while in flight. B) At all times when the OAT is below freezing. C) Whenever icing conditions are first encountered or are expected to occur. 40 PLT117 FEX In an electrically heated windshield system, what senses normal windshield temperature? A) Thermistors. B) Electronic amplifiers. C) Thermal overheat switches. 41 PLT206 FEX What is the highest ambient temperature that ice is likely to form in the engine inlet? A) Visibly moist air and +45 °F. B) Visibly moist air and +70 °F. C) Relatively dry air and +32 °F. 42 PLT190 FEX Which is an indication of carburetor ice with engines having constant speed propellers? A) Decrease in propeller RPM. B) Manifold pressure (MAP) drop. C) Backfiring, which is caused by a rich mixture. 43 PLT109 FEX Overcharging a nickel-cadmium battery will likely result in a release of A) toxic nickel hydroxide liquid or steam. B) oxygen and explosive hydrogen gases. C) highly combustible acetylene and oxygen gases. 44 PLT109 FEX Thermal runaway in nickel-cadmium batteries are usually caused by? A) Low temperatures and high discharge rates. B) Deep rapid discharges and low charge rates. C) High temperatures and constant-voltage charging. 45 PLT109 FEX What causes thermal runaway in a nickel-cadmium battery? A) Low temperatures and high discharge rates. B) Deep rapid discharges and low charge rates. C) High temperatures and constant-voltage charging. 46 PLT109 FEX What condition characterizes a thermal runaway? A) Increased resistance of the battery to input current. B) High temperature and undercharging at a constant rate. C) Continuous rising current and increasing battery temperature. 47 PLT109 FEX What instrument readings would indicate a ni-cad battery thermal runaway? A) Increasing OAT and generator loading while the battery charge current is decreasing. B) High temperature and battery discharging at a constant rate, regardless of loadmeter indications. C) Continuously rising charge current and increasing battery temperature. 48 PLT109 FEX What type of gas is released by a nickel-cadmium battery during overcharging? A) Toxic nickel hydroxide. B) Oxygen and explosive hydrogen. C) Highly combustible acetylene and oxygen. 49 PLT207 FEX Why is it important that all electrical loads and power sources be turned off before connecting or disconnecting the battery? A) To prevent discharging the battery. B) To prevent a spark from igniting explosive gas. C) To prevent power surges from spiking sensitive equipment. 50 PLT207 FEX A relay usually functions as a remote switch, whereas a solenoid A) must have AC power at all times to function. B) can actuate remote valves or switches. C) functions as a gang valve assembly. 51 PLT207 FEX Fuses are sized to carry no more current than A) the generator will energize. B) the wires can carry. C) a holder will use. 52 PLT207 FEX How many spare electrical fuses are required for use in flight? A) One complete spare set. B) Three fuses of each size that is installed. C) The number approved for that airplane described in the certificate holder`s manual. 53 PLT207 FEX In aircraft electrical systems, automatic reset circuit breakers A) are not used as circuit protective devices. B) are used in all circuits essential to safe operation of the aircraft. C) are found in locations where only temporary overloads are encountered. 54 PLT207 FEX Thermal protectors are used to A) stop windshield heaters from melting the glass. B) protect motors from overheating. C) allow pitot heaters to melt any icing near the tube. 55 PLT207 FEX What is the difference between a relay and a solenoid? A) Relays have movable cores. B) Solenoids have movable cores. C) Relays are used as mechanical control devices. 56 PLT207 FEX What is the purpose of a fuse-type current limiter? A) Prevent overloads in low power circuits. B) Fast blow design prevents damage to sensitive circuits or equipment. C) Permit short periods of overload before the fuse link melts and breaks the circuit. 57 PLT207 FEX An ac generator control unit will A) protect the generator and electrical system from reverse currents, overexcitation, and overvoltages. B) convert undervoltage to the desired value, acknowledge differential faults, and facilitate manual paralleling. C) command generator rpm increases in underspeed conditions and include bus-tie circuit-breaker automatic closing. 58 PLT207 FEX If one of the generators fails in a parallel bus electrical system, A) bus control circuitry will automatically shed all of that generator`s load. B) the electrical load is automatically supplied by the remaining generator, up to it`s load limit. C) Each generator supplies power separately from the other generators to its respective bus. 59 PLT207 FEX The purpose of a KVAR meter is to A) display the generator frequency. B) measure the work being performed. C) indicate how hard the generator is working to produce the power being used. 60 PLT207 FEX What increases or decreases the voltage of a generator so it carries its share of the load? A) Current limiter. B) Paralleling circuit. C) Reverse current cut-out relay. 61 PLT207 FEX What is residual voltage? A) Voltage produced that is not in phase with the current. B) Voltage stored in the generator exciter output windings. C) Voltage produced by permanent magnets which starts the ac generator output. 62 PLT207 FEX What speed does a frequency meter give a direct indication of? A) Engine N2. B) Generator RPM. C) CSD input speed. 63 PLT207 FEX What type voltage will be produced if an ac generator is being driven, but there is no field excitation? A) Real voltage. B) Residual voltage. C) Reactive voltage. 64 PLT207 FEX Which are protective functions of an ac generator control unit? A) Reverse current, overexcitation, and overvoltage. B) Undervoltage, differential fault, and manual paralleling. C) Generator underspeed and bus-tie circuit-breaker automatic closing. 65 PLT207 FEX Which is a feature of a parallel bus electrical system? A) External power may be paralleled with operating generators. B) The electrical load is automatically redistributed when one generator fails. C) Each generator supplies power separately from the other generators to its respective bus. 66 PLT207 FEX Which is a purpose of a transformer rectifier? A) Converts 115 volts ac, 400-Hz to 28 volts dc. B) Changes dc to alternating 26 volts or 115 volts, 400-Hz power. C) Operates emergency flight instruments and radios from the airplane battery. 67 PLT207 FEX A thermocouple is A) a pair of dissimilar wires connected together. B) made to detect light. C) able to sense unsafe vibrations. 68 PLT207 FEX What is an advantage of using 115 volts, 400-Hz alternating current? A) High voltage and low current reduces wire size and weight requirements. B) Commutators may be used with ac motors to decrease repair intervals and costs. C) The ac single-phase induction motors are self-starting, making it possible to use lightweight motors. 69 PLT207 FEX What is a purpose of electrical bonding jumpers? A) Decrease the probability of lightning damage to such elements as control hinges. B) Minimize electrolytic corrosion by connecting the airplane parts to form an integral unit. C) Provide a high-resistance path for electrical equipment, thereby eliminating ground wires. 70 PLT327 FEX What is the indication of a thermal discharge of a gaseous oxygen system? A) The blowout disk is ruptured. B) The pressure gauge indicates zero. C) The heat sensitive paint marks change from white to black. 71 PLT326 FEX What is an advantage of a chemical over a gaseous oxygen system? A) Fire hazards are reduced by eliminating oxygen lines. B) Chemical systems may be shutoff at any time after they are activated. C) Reliability is improved by interconnecting individual chemical units. 72 PLT327 FEX What is one danger of any oxygen leak? A) Oxygen being highly flammable may cause combustible materials to burn intensely. B) Combustible materials will ignite more rapidly and burn with greater intensity in oxygen rich conditions. C) Any ignition source may ignite highly explosive oxygen which over a period of time saturates the surroundings in poorly vented areas. 73 PLT326 FEX What oxygen flow condition should exist if the oxygen regulator selector is placed in the emergency position and the supply lever is on? A) 100 percent oxygen available on demand. B) Continuous flow of diluted oxygen under positive pressure. C) Continuous flow of 100 percent oxygen under positive pressure. 74 PLT326 FEX What type of oxygen system is used for passengers? A) Demand. B) Constant-flow. C) Diluter-demand. 75 PLT326 FEX When should 100% oxygen be selected for flight deck masks? A) During all flight time above 10,000 feet. B) When first applying the mask. C) To correct a feeling of lack of oxygen. 76 PLT135 FEX If the cabin rate of climb is too great, how should the pressurization controls be adjusted? A) Open the outflow valve slower. B) Close the outflow valve faster. C) Increase the amount of incoming air. 77 PLT135 FEX The cabin pressure control setting has a direct effect upon the A) compressor speed. B) outflow valve opening. C) pneumatic system pressure. 78 PLT135 FEX Which best describes cabin differential pressure? A) The difference between ambient and internal air pressure. B) The difference between the cabin flight altitude pressure and MSL pressure. C) The difference between the cabin pressure controller setting and the actual cabin pressure. 79 PLT135 FEX Which control systems for operating cabin pressurization use reference chamber air pressure within the controller to regulate the outflow valve? A) Isobaric and differential. B) Unpressurized and pressurized controls. C) Ambient, differential, and maximum differential. 80 PLT137 FEX Which cabin air-conditioning system utilizes a refrigerant to carry away cabin heat? A) Air-cycle. B) Vapor cycle. C) Evaporative blower. 81 PLT137 FEX Which components make up the basic air-cycle cooling system? A) Heaters, coolers, and compressor. B) Ram air source, compressors, and engine bleeds. C) A source of compressed air, heat exchangers, and a turbine. 82 PLT212 FEX From a standpoint of toxicity and corrosion hazard, which fire extinguishing agent is safest to use in turbojet airplanes? A) Carbon dioxide. B) Methyl bromide. C) Chlorobromomethane. 83 PLT212 FEX On a built-in carbon dioxide fire extinguishing system, how is a thermal discharge detected? A) The thermal plug is missing from the side of the bottle. B) The red plastic disc in the thermal discharge line is missing. C) The yellow plastic disc in the thermal discharge line is discolored. 84 PLT212 FEX What is the perferred fire extinguishing agent for installed fire extinguishing systems? A) Halon 1001. B) CO2. C) Halon 1211. 85 PLT212 FEX Which indicates that a fire extinguishing system has been intentionally discharged into a turbojet engine? A) Missing red disc on the side of the fuselage. B) Missing green disc on the side of the fuselage. C) Missing yellow disc on the side of the fuselage. 86 PLT139 FEX How does the thermocouple in a fire detection system cause the warning system to operate? A) Heat increases electrical resistance. B) Heat generates a small electrical current. C) Heat causes expansion and a ground to form. 87 PLT139 FEX The optical smoke detectors on the flight engineer panel correspond to A) light beam responses to cargo hold air samples. B) closed circuit mini-cam installations in the cargo hold. C) translucent spotter tubes providing a view of the cargo holds. 88 PLT139 FEX When an airplane is equipped with a continuous-loop fire detection system, which is the most common cause of false fire warnings? A) Moisture in the system. B) Worn clamps, vibration and chafing of sensor loops. C) Improper routing or connection of detector loops. 89 PLT139 FEX Which type of fire detector circuit can continue to function with either one open or one short in the detector loops? A) Continuous loop. B) Two-wire thermal switch. C) Single-wire thermal switch. 90 PLT346 FEX Airplanes equipped with both inboard and outboard ailerons normally use the outboard ailerons only during A) low-speed operations. B) high-speed operations. C) low-altitude operations. 91 PLT346 FEX How does an aileron balance panel function? A) A weight is installed ahead of the hinge line to counteract flight loads. B) The aileron is extended ahead of the hinge line so the airstream will help move the surface. C) Pressure changes created by the aileron deflect a hinged panel in a compartment ahead of the aileron. 92 PLT473 FEX The purpose of an antiservo tab is to A) move the flight controls in the event of manual reversion. B) reduce control forces by deflecting in the proper direction to move a primary flight control. C) prevent a control surface from moving to a full deflection position due to aerodynamic forces. 93 PLT473 FEX Which direction from the primary control surface does an antiservo tab move? A) Same direction. B) Opposite direction. C) Remains fixed for all positions. 94 PLT473 FEX The purpose of a servo tab is to A) allow lower control pressures to move large main flight controls. B) induce flight control feedback forces by deflecting in the proper direction when a primary flight control surface moves. C) prevent a control surface from moving to a full deflection position due to aerodynamic forces. 95 PLT473 FEX Which direction from the primary control surface does a servo tab move? A) Same direction. B) Opposite direction. C) Remains fixed for all positions. 96 PLT473 FEX The purpose of an elevator trim tab is to A) provide horizontal balance as airspeed is increased to allow hands-off flight. B) adjust the upward tail load for different airspeeds in flight allowing neutral control forces. C) modify the downward tail load for various airspeeds in flight eliminating flight control pressures. 97 PLT278 FEX What information does a Mach meter present? A) The ratio of airplane true airspeed to the speed of sound. B) The ratio of airplane indicated airspeed to the speed of sound. C) The ratio of airplane equivalent airspeed, corrected for installation error, to the speed of sound. 98 PLT253 FEX One purpose of a fuel tank boost pump is to prevent vapor lock caused by low A) temperature. B) altitude operation. C) atmospheric pressure. 99 PLT253 FEX The fuel heater system may be A) manually or automatically controlled. B) on and heating all the time. C) on and heating all the time with JET A fuel. 100 PLT253 FEX Which is a means of controlling the fuel temperature on turbojet-powered airplanes? A) Electrically heated fuel filters. B) Engine bleed air routed to a heat exchanger. C) Fuel filters heated by engine lubricating oil. 101 PLT250 FEX Most AVGAS is A) purple. B) red. C) blue. 102 PLT115 FEX What will result if an insufficient amount of ADI fluid is injected during takeoff? A) Temporary power increase. B) Engine overheat and detonation. C) Power will remain the same if the fuel/air ratio is increased. 103 PLT251 FEX What is the difference between turbine fuel Jet A and Jet A-1? A) Jet A is made for operation at extremely low temperatures. B) Jet A-1 is made for operation at extremely low temperatures. C) Jet A is for use in older turbine aircraft, while Jet A-1 is formulated for the newest aircraft. 104 PLT324 FEX For engines equipped with Hamilton-Standard Hydromatic propellers the purpose of feeding the engine oil pressure pump from a standpipe is to A) minimize the amount of oil that has to be diluted with fuel in cold weather. B) provide oil for feathering the propeller in the event an oil line should break. C) separate the circulating oil from the surrounding oil when the engine is started to permit a fast warmup of the engine. 105 PLT324 FEX Oil extracts the most heat from which turbine engine components? A) Turbine bearings. B) Compressor bearings. C) Accessory drive bearings. 106 PLT324 FEX The purpose of the hopper tank is to A) permit a fast warmup of the engine oil. B) collect sludge and particles from the oil in the event the oil filter becomes obstructed. C) provide engine oil to feather the propeller in the event an oil line should break and all of the engine oil is pumped overboard. 107 PLT324 FEX If the oil cooler relief valve should stick in the open position, what would be the probable result? A) Decreased oil temperature. B) Increased oil temperature. C) Pressurization of the case and oil leakage. 108 PLT251 FEX Mixing aviation gasoline with jet fuel will effect a turbine powerplant by forming deposits on the A) turbine blades. B) compressor blades. C) inlet guide vanes. 109 PLT251 FEX What is the difference between Jet A and Jet B fuel? A) Jet A is colorless and Jet B is straw colored. B) Jet A has a freeze point of -40 °C and Jet B has a freeze point of -47 °C. C) Jet B is a blend of gasoline and kerosene made for operating at extremely low temperatures. 110 PLT324 FEX What is the oil viscosity grade? The oil viscosity grade A) indicates how fluid an oil is at low temperature under laboratory conditions. B) is an arbitrary method of stating the rate of change in viscosity of an oil with changes of temperature. C) is the weight of any oil compared with the weight of an equal volume of oil from the American Petroleum Institute (API) gravity scale. 111 PLT251 FEX Why are jet fuels more susceptible to water contamination than aviation gasoline? A) Jet fuel has a higher viscosity than gasoline. B) Jet fuel is lighter than gasoline, and suspends water easier. C) Condensation is greater because of large temperature changes at high altitudes. 112 PLT273 FEX An advantage of a piston-type accumulator is that it A) takes up less area than a sphere-type accumulator. B) may be used with higher pressure than a bladder-type accumulator. C) can store more hydraulic fluid than a diaphragm-type accumulator. 113 PLT273 FEX The purpose of a hydraulic accumulator is to A) store hydraulic fluid under pressure. B) collect hydraulic fluid from system leaks. C) gather foam and extract the air before returning it to the reservoir. 114 PLT273 FEX What type of gas may be used to service hydraulic accumulators? A) Nitrogen. B) Dry oxygen. C) Carbon dioxide. 115 PLT273 FEX What will the hydraulic pressure gauge indicate when the accumulator charge is lost and the monitor is on the air side of a diaphragm-type accumulator? A) Zero. B) System pressure. C) Between one-third and one-half of the system's operating pressure. 116 PLT273 FEX What is the significance of the pop-out indicator on some hydraulic filters? A) Confirmation of fluid thermal expansion in the gear, flaps, or other systems. B) Evidence that contaminants may prevent components such as check valves from operating. C) Indication of overfilling and the fluid level of the hydraulic reservoir should be checked. 117 PLT273 FEX Why should hydraulic fluid be filtered? A) Water in the fluid could freeze. B) It assures a positive feed of foam free fluid to the hydraulic pump inlet. C) Contaminants may damage the seals and cylinder walls causing internal leakage. 118 PLT273 FEX What color identifies MIL-H-5606? A) Red. B) Amber. C) Green. 119 PLT273 FEX How may pressure from an engine-driven hydraulic pump be regulated? A) Constant speed drive. B) Variable-displacement pump. C) In-line variable restrictor orifice. 120 PLT273 FEX The purpose of pressurizing a hydraulic reservoir is to A) provide an alternate source of pressure in case of a hydraulic pump failure. B) assure a positive feed of foam free fluid to the hydraulic pump at high altitudes. C) insure an adequate supply of fluid to the hydraulic pump inlet during negative-G flight. 121 PLT273 FEX What is the difference between a hydraulic sequence valve and a priority valve? A) Sequence valves are electrically actuated. B) Mechanical contact opens a priority valve. C) Priority valves are opened by hydraulic pressure. 122 PLT110 FEX The main purpose of a brake debooster is to A) provide pneumatic braking. B) reduce the hydraulic pressure. C) decrease the volume of fluid going to the brakes. 123 PLT110 FEX The purpose of the antiskid system control box is to A) sense wheel speed change. B) prevent landing with the brakes applied. C) meter the brake pressure to prevent stoppage of wheel rotation. 124 PLT110 FEX What item of the antiskid system enables full braking action during taxi and parking? A) The locked-wheel arming circuit is inoperative at taxi speeds. B) The antiskid switch is manually disarmed on the afterlanding checklist. C) The wheel-speed sensors automatically disarm the antiskid system at speeds below about 20 miles per hour. 125 PLT139 FEX What safety device is actuated by the compression and extension of a landing gear strut? A) Uplock switch. B) Downlock switch. C) Ground safety switch. 126 PLT139 FEX When will the landing gear position warning system provide a warning in the cockpit? A) When the power levers are retarded and the gear is in transit. B) When the power levers are advanced and the speed brakes are not retracted. C) When one power lever is retarded below cruise and the gear is down and locked. 127 PLT138 FEX What tread wear will occur if tires are underinflated? A) Uniform wear at a fast rate. B) Tread worn away more on the shoulders than in the center. C) Accelerated centerline wear while leaving rubber on the shoulder. 128 PLT138 FEX The purpose of chines on tires is to A) increase traction on snow or ice covered runways. B) reduce the tendency to hydroplane on wet runways. C) deflect water or slush away from the engine intakes. 129 PLT138 FEX When inflating tires from a high pressure bottle, always A) use industrial oxygen. B) use a pressure regulator on the bottle. C) inflate directly from the nitrogen bottle slowly. 130 PLT138 FEX The purpose of fusible plugs in aircraft wheels is to A) prevent tire blowouts. B) quickly deflate tires for repair. C) protect the antiskid electrical system. 131 PLT338 FEX Moisture in a pneumatic system is removed A) by the moisture separator. B) in the compressor stage. C) by the filter. 132 PLT115 FEX What may cause engine detonation? A) High octane fuel. B) Low manifold pressure. C) Excessively lean fuel-air mixture. 133 PLT499 FEX (Refer to figure 10.) What is the indicated speed of the engine low pressure compressor? A) 94.1 percent. B) 96.5 percent. C) 97.0 percent. 134 PLT365 FEX The BMEP indicator measures the A) ratio of the shaft output to the power developed in the cylinders. B) output shaft torque and converts it to BMEP. C) actual power output to the propeller by converting heat energy to mechanical energy. 135 PLT365 FEX The indicated horsepower of a reciprocating engine is defined by the A) computed horsepower based on engine RPM and manifold pressure adjusted to sea level. B) power developed in the combustion chambers less computed friction losses within the engine. C) power developed in the combustion chambers without reference to friction losses within the engine. 136 PLT209 FEX What will the EPR indication be in the case of iced over EPR probes? A) The Pt probe vent will act as a Ps probe. B) The EPR gauge will indicate a false high value. C) The reduced inlet area causes pressure to increase, and magnifies the influence of ram air pressure. 137 PLT499 FEX Which engine instrument is most likely to show an elevated reading if the turbine wheel is damaged? A) N2. B) EPR. C) EGT or TIT. 138 PLT499 FEX Which engine instrument will indicate a higher-than-normal reading if the compressor has damage? A) Engine RPM. B) Torquemeter. C) Turbine inlet temperature. 139 PLT499 FEX (Refer to figure 9.) Combustion takes place in which location? A) Location 2. B) Location 4. C) Location 7. 140 PLT210 FEX The purpose of shutting an engine down with the mixture control at the end of the flight is to A) prevent an accidental start. B) preclude liquid lock during subsequent starts. C) assure that there is no fuel in the intake system that could result in a fire. 141 PLT210 FEX Why should turbine engines normally be operated at idle for a period of time before shutdown? A) The turbine case cools faster and may shrink down and seize the turbine blades. B) Rapid cooling of the compressor section may cause cracking of compressor blades. C) Temperature reduction and stabilization prevents a hot combustion chamber from igniting residual fuel. 142 PLT343 FEX Excessive oil in radial engines in the the lower cylinders between pistons and heads is an indication of A) worn oil control rings. B) oil supply line bypass valve failure. C) intercylinder drains that are partially or completely blocked. 143 PLT479 FEX While starting a turbine engine with an air starter, a hung start occurs before the starter disengages. Which procedure is correct? A) Shut down the engine. B) Increase the air velocity to the starter. C) Slowly increase the power lever until the engine accelerates to idle. 144 PLT342 FEX The primary purpose of exhaust augmenters is to A) increase engine cooling. B) provide additional thrust. C) decrease exhaust back pressure. 145 PLT124 FEX What effect does an increase in atmospheric humidity have on brake horsepower output of a water/alcohol injected engine? A) A power loss will be experienced by either a wet or dry takeoff. B) A wet engine takeoff will lose power more rapidly than a dry engine takeoff. C) A pressure-injected carburetor will not be affected by increased humidity. 146 PLT478 FEX Does placing the magneto switches in the OFF position guarantee that the propellers are safe to handle? A) No, the only safe way to ensure the engines will not fire accidently is to disconnect the battery. B) No, the magneto switches may fail in the closed position and current will continue to be supplied to the ignition system. C) No, to turn off the ignition the magnetos operate on the principle of short-circuiting the current and a loose ground wire can allow a cylinder to fire. 147 PLT115 FEX Afterfiring is caused by A) the spontaneous combustion of the unburned charge ahead of the flame fronts after the ignition of the charge. B) charges of unburned fuel in the exhaust gas mixing with air outside the exhaust and igniting, causing an explosion in the exhaust system. C) a mixture so lean that combustion not completed on the exhaust stroke ignites the contents of the intake manifold when the intake valve opens. 148 PLT134 FEX A minimum loss of power will occur on takeoff in high humidity conditions if A) the carburetor is set at full rich. B) the mixture is set for emergency rich and additional fuel is injected. C) the fuel/air ratio is set for best power and the takeoff is wet (with antidetonation fluid). 149 PLT115 FEX Backfiring is caused by A) fouled spark plugs, defective fuel injection nozzles, or incorrect valve clearances. B) charges of unburned fuel in the exhaust gas mixing with air outside the exhaust and igniting, causing an explosion in the exhaust system. C) a mixture so lean that combustion not completed on the exhaust stroke ignites the contents of the intake manifold when the intake valve opens. 150 PLT249 FEX What does an increase of approximately 125 propeller RPM indicate during shutdown? A) Idle mixture is correct. B) Idle mixture is too lean. C) Idle mixture is too rich. 151 PLT479 FEX If an attempt is made to start an engine with a hydraulic lock, A) a connecting rod can bend or break if the crankshaft continues to rotate. B) the starter gearbox can overtorque since the liquid is incompressible and stops piston movement. C) the fuel or oil from the lower cylinders can be injected into the exhaust system causing afterfiring. 152 PLT365 FEX How are the cylinders numbered in a double-row radial engine? A) Clockwise as viewed from the engine front. B) Clockwise as viewed from the accessory end. C) Counterclockwise as viewed from the accessory end. 153 PLT479 FEX During starting, what should prevent the engine from driving a pneumatic starter to burst speed? A) Drive shaft shear point. B) Sprag clutch assembly. C) Design of the starter turbine nozzle vanes which causes choking. 154 PLT478 FEX Why are pneumatic starters used on most large turbine engines? A) Less weight. B) Simple design requires no clutch. C) Air starters are mechanically more reliable than electrical starters. 155 PLT479 FEX During engine start, closing of the start air valve may be verified by A) engine RPM stabilizing at idle. B) air manifold pressure increasing slightly. C) air manifold pressure decreasing slightly. 156 PLT499 FEX It is important to monitor the EGT when starting the engines to prevent A) hot section burnout or metal distortion. B) compressor temperature limits from being exceeded. C) dangerous gas temperatures and velocities when accelerating to idle from injuring personnel or damaging the engine. 157 PLT499 FEX Which is the most critical parameter for a turbine engine during starting? A) TIT. B) Oil pressure. C) Starter engagement time. 158 PLT343 FEX While performing the ground check the supercharger control is shifted from the high to the low position. Normal operation of the supercharger will be indicated by A) a sudden decrease in manifold pressure. B) the manifold pressure remaining the same. C) a momentary increase in manifold pressure. 159 PLT499 FEX Equivalent shaft horsepower (ESHP) is defined as A) the total power delivered to the propeller. B) the shaft horsepower plus the effects of the jet thrust produced by the engine. C) the actual amount of horsepower delivered to the propeller shaft that is equivalent to 33,000 foot-pounds of work per minute. 160 PLT499 FEX Exceeding the engine temperature limitations may result in A) discoloration of the compressor blades. B) rippling of the trailing edge of the compressor blades. C) hairline cracks at right angles to the turbine blade leading and trailing edges. 161 PLT499 FEX Heat and centrifugal force on turbine blades causes A) galling. B) creeping. C) stretching. 162 PLT499 FEX How does compressor RPM affect the power output of a turbofan engine? A) Power increases linearly with an increase in compressor speed. B) Efficiency increases when compressor blade tips reach Mach 1 or slightly higher. C) Compressor aerodynamics cause a nonlinear increase in power relative to compressor speed. 163 PLT499 FEX On most engines ram recovery occurs above A) 60 knots. B) 100 knots. C) 140 knots. 164 PLT499 FEX Which location has the highest gas volume in a turbine engine? A) Turbine outlet. B) Compressor outlet. C) Combustion chamber outlet. 165 PLT237 FEX Why is 36,000 feet considered an optimum altitude for most airplanes during long range flights? A) The range between high and low speed Mach buffet decreases rapidly above this altitude. B) Decreased aerodynamic drag is not as great as the decrease in engine thrust above this altitude. C) Above this altitude the increase in drag from high angles of attack is greater than the increase in engine thrust. 166 PLT499 FEX (Refer to figure 7.) The internal engine pressure will be the lowest at which location? A) Location 4. B) Location 5. C) Location 6. 167 PLT499 FEX (Refer to figure 7.) Which engine location will have the highest internal engine temperature? A) Location 3. B) Location 4. C) Location 6. 168 PLT499 FEX (Refer to figure 8.) Where is the engine high pressure compressor located? A) Location 1. B) Location 2. C) Location 3. 169 PLT499 FEX In a dual axial-flow compressor system the first stage turbine drives the A) N1 compressor. B) N2 compressor. C) N1 and N2compressors. 170 PLT499 FEX The function of the nozzle diaphragm located on the upstream side of the turbine wheel is to A) increase the pressure of the exhaust mass. B) increase the velocity of the heated gases flowing past the nozzle diaphragm. C) decrease the velocity of the heated gases flowing past the nozzle diaphragm. 171 PLT499 FEX The purpose of the diffuser section in a turbine engine is to A) convert pressure to velocity. B) increase pressure and reduce velocity. C) reduce pressure and increase velocity. 172 PLT499 FEX The speed (RPM or percent) of the innermost compressor of a triple-spool turbofan engine is referred to as A) N1. B) N2. C) N3. 173 PLT499 FEX The speed (RPM or percent) of the low pressure compressor of a dual compressor engine is referred to as A) N1. B) N2. C) N3. 174 PLT499 FEX The two basic elements of the turbine section of a turbine engine are the A) rotor and stator. B) bucket and expander. C) impeller and diffuser. 175 PLT499 FEX Which location has the highest gas pressure in a turbine engine? A) Turbine outlet. B) Compressor outlet. C) Combustion chamber outlet. 176 PLT210 FEX Why should shutting an engine down by turning the boost pumps off and closing the airplane`s fuel valve be used during emergencies only? A) During a subsequent start the engine driven fuel pump may flood the combustion chamber or burner can. B) The fuel system`s service life will be decreased because fuel wetted components will not be lubricated and may lose their prime. C) The 30 to 60 seconds for the fuel to clear the fuel lines from the firewall to the fuel controller is too long for normal shutdowns. 177 PLT499 FEX How does a change in altitude affect the low pressure compressor speed of a triple-spool engine? A) The compressor will speed up as altitude is increased. B) The fuel control will maintain a constant compressor speed. C) The compressor speed will decrease as the atmosphere becomes less dense. 178 PLT499 FEX The speed (RPM or percent) of the high pressure compressor of a dual compressor engine is referred to as A) N1. B) N2. C) N3. 179 PLT499 FEX (Refer to figure 7.) The internal engine temperature will be the lowest at which location? A) Location 3. B) Location 4. C) Location 6. 180 PLT499 FEX What does Pt7 mean? A) Total inlet pressure at Station no. 7. B) Total absolute pressure at Station no. 7. C) Pressure and temperature at Station no. 7. 181 PLT499 FEX Which difference does engine pressure ratio measure? A) Uncorrected compressor inlet pressure and turbine discharge pressure. B) Compressor inlet total pressure and turbine discharge total pressure. C) Compressor outlet total pressure corrected for temperature and turbine discharge total pressure. 182 PLT479 FEX If a turbine engine catches fire internally during the start cycle, what initial action should be taken? A) Secure all switches and leave the airplane. B) Shut off the fuel and continue motoring the starter. C) Discharge CO2 into the inlet duct while continuing to motor the engine. 183 PLT343 FEX Power is increased by the turbines in a turbocompound engine by A) exhaust gas velocity powering blow-down turbines geared to the crankshaft. B) engine driven turbines compressing the fuel/air mixture after it leaves the carburetor to increase the manifold pressure. C) exhaust gas pressure driving power-recovery turbines which compress the air before it is mixed with metered fuel from the carburetor. 184 PLT351 FEX Beta range refers to the A) propeller RPM range controlled by the prop levers. B) propeller blade angle which can produce zero or negative thrust. C) second letter of the Greek alphabet used to represent a constant propeller RPM within the flight range of the throttle. 185 PLT351 FEX Blade angle is the angle between the A) airfoil chord line and the relative wind. B) propeller's airfoil chord line and its plane of rotation. C) face of the propeller blade and the direction of the relative airstream. 186 PLT351 FEX In the propeller deicing system, electrical power is transferred to the propeller hub assembly A) through slip rings and carbon brushes. B) through flexible electrical connectors. C) by use of slip rings and segment plates. 187 PLT351 FEX Feathering of a Hamilton-Standard Hydromatic propeller can be accomplished by A) pushing in the feather button. B) pulling the fire emergency control handle. C) moving the propeller control lever to the full aft position. 188 PLT351 FEX The force which tries to feather the propeller blade is A) torque bending. B) aerodynamic twisting. C) centrifugal twisting moment. 189 PLT351 FEX Which best describes the blade movement of a feathering propeller set in the HIGH RPM position when the feathering action is begun? A) Low pitch through reverse, to feather. B) High pitch through low pitch, to feather. C) Low pitch through high pitch, to feather. 190 PLT351 FEX (Refer to figure 11.) The propeller condition depicted is A) onspeed. B) overspeed. C) underspeed. 191 PLT351 FEX Retarding the throttle/power setting in cruise flight will result in A) a decrease in blade angle. B) an increase in blade angle. C) a decrease in propeller RPM. 192 PLT351 FEX The principle which operates a Hamilton-Standard Hydromatic propeller is oil pressure A) to decrease or increase the blade angle. B) to decrease the blade angle and counterweights to increase the blade angle. C) and centrifugal twisting moment to decrease the blade angle and, counterweights and springs to increase the blade angle. 193 PLT351 FEX Which operational force creates the greatest stress on a propeller? A) Centrifugal. B) Torque bending. C) Aerodynamic twisting. 194 PLT351 FEX The function of the propeller synchrophase system is to A) set the blades of all propellers to an identical blade angle. B) maintain slave engine RPM within 3 percent of the master engine RPM. C) maintain a predetermined angular relationship between the blades of all propellers as they rotate. 195 PLT351 FEX What initial action is taken to unfeather a Hamilton-Standard Hydromatic propeller? A) Place the aircraft in a shallow dive to start the propeller windmilling. B) Turn the autofeather system off and place the propeller lever to the full forward position. C) Hold the feather button in until the propeller starts windmilling, then release for restart. 196 PLT108 FEX What is the minimum glycol content of Type 1 deicing/anti-icing fluid? A) 30 percent. B) 50 percent. C) 80 percent. 197 PLT108 FEX Which of the following procedures will increase the holding time during the anti-ice phase of a two-step process? A) Glycol content is raised to 100 percent. B) The Type 2 fluid is heated before application. C) The Type 2 fluid is applied with centrifugal pumps. 198 PLT108 FEX The purpose of diluting ethylene glycol deicing fluid with water in nonprecipitation conditions is to A) raise the eutectic point. B) decrease the freeze point. C) increase the minimum freeze point (onset of crystallization). 199 PLT108 FEX What precaution should be taken when using truck-mounted deice/anti-ice equipment? A) Run the airplane engines at idle. B) Spray engine and APU inlets directly. C) Spray pitot inlets and static ports indirectly. 200 PLT108 FEX What safeguard should be taken when using mobile ground deice/anti-ice equipment? A) Open the air-conditioning pack valves. B) Operate the airplane engines above idle to prevent flameout. C) Apply fluid to the lower door sills and the door bottoms prior to closing for flight. 201 PLT108 FEX Anti-icing fluid should provide freezing point protection to A) -20 °F ambient temperature. B) +32 °F outside temperature or below. C) a freezing point no greater than 20 °F below the ambient or airplane surface temperature. 202 PLT108 FEX Deicing fluid should be dispensed at what temperature? A) Cold. B) Heated. C) Ambient. 203 PLT108 FEX What determines the viscosity of Type 1 deicing/anti-icing fluid? A) Temperature. B) Thickening agents. C) Dispensing equipment. 204 PLT108 FEX What should the deice/anti-ice fluid temperature be during the last step of a two-phase process? A) Hot. B) Warm. C) Cold. 205 PLT108 FEX Type 2 deicing/anti-icing fluids have a significantly A) longer holdover period than type 1 fluids. B) shorter holdover period than type 1 fluids. C) lower viscosity than type 1 fluids. 206 PLT212 FEX What identifies a fire extinguisher used for brake fires? A) A square with the letter B. B) A circle with the letter C. C) A star with the letter D. 207 PLT208 FEX Which frequency is preferred to declare an emergency to ATC? A) 121.5 Mhz VHF. B) 243.0 Mhz UHF. C) The one in use. 208 PLT212 FEX Which is the most effective extinguishing agent for use on an electrical fire? A) Carbon dioxide. B) Methyl bromide. C) Carbon tetrachloride (Halon 04). 209 PLT497 FEX Which transponder code means the airplane is being forced to a new destination? A) 7500 B) 7600 C) 7700 210 PLT318 FEX What does declaring minimum fuel to ATC imply? A) Traffic priority is needed to the destination airport. B) Emergency handling is required to the nearest useable airport. C) An emergency situation is possible should an undue delay occur. 211 PLT326 FEX To which position should the oxygen regulator be set when symptoms of hypoxia or hyperventilation are experienced? A) Normal. B) Emergency. C) 100 percent oxygen. 212 PLT497 FEX What altitude information is transmitted by MODE C of the transponder? A) Pilot's indicated altitude. B) Altitude in 10-foot increments. C) Altitude without barometric pressure correction. 213 PLT389 FEX Which CFR governs airplane operations when common carriage is not involved with airplanes over 80,000 pounds GTOW or more than 20 passenger seats? A) CFR part 135 for cargo flights. B) CFR part 121 for test flights. C) CFR part 125 for passenger flights. 214 PLT388 FEX For what purpose may information obtained from cockpit voice recorders and flight data recorders not be used? A) Identifying malfunctions and irregularities in aircraft systems. B) Determining causes of accidents and occurrences under investigation by the National Transportation Safety Board (NTSB). C) Determining any certificate action or civil penalty arising out of an accident or occurrence. 215 PLT440 FEX What is considered to be flight crewmember essential conversation? A) Discussing the weather north of the flight route. B) Discussing the crew configuration for the following leg of the flight. C) Confirming airplane logbook entries during climb when clear of the airport traffic area. 216 PLT440 FEX Which flight crewmember nonessential conversation is allowed below 10,000 feet? A) Discussing stock market reports during taxi. B) None. C) Confirming airplane logbook entries during climb when clear of the airport traffic area. 217 PLT388 FEX How long shall cockpit voice recorder and flight recorder data be kept in the event of an accident or occurrence resulting in termination of the flight? A) 60 days. B) 90 days. C) 120 days. 218 PLT409 FEX What is the flight time limitation for flag operations that requires two pilots and at least one additional flight crewmember? A) 100 hours during any 30-day period. B) 300 hours during any 3 calendar months. C) 1,000 hours during any 12 calendar-month period. 219 PLT405 FEX Each crewmember shall have available for individual use on each flight a A) quick-donning type oxygen mask. B) flashlight in good working order. C) hand fire extinguisher suitable for extinguishing Class A, B, and C fires. 220 PLT388 FEX Which factors must be recorded by the approved flight recorder? A) Airspeed, time, altitude, vertical acceleration, and heading. B) Time, true altitude, calibrated airspeed, vertical speed, and heading. C) Elapsed time, airspeed, altitude, vertical acceleration, and magnetic course. 221 PLT386 FEX A crewmember certificate may be issued by the FAA to flight crewmembers on U.S. registered aircraft engaged in A) international air commerce. B) flight crewmember training only. C) supplemental air carrier operations. 222 PLT410 FEX What are the minimum flight engineer operating experience requirements for reciprocating-powered airplanes when common carriage is involved? A) Flight engineer duties performed for 8 hours under the supervision of a check airman in flight. B) Flight engineer duties performed for 10 hours under the supervision of a qualified flight engineer. C) Flight engineer duties performed for 12 hours under the supervision of a qualified pilot in command. 223 PLT451 FEX Which requirement must be met by all flight engineers every 6 months before they can serve on an air carrier flight under 14 CFR part 121? A) Line check or route check. B) Recurrent flight and ground training. C) 50 hours of flight time or a flight check. 224 PLT407 FEX A flight engineer operating under CFR part 121 must receive recurrent training on A) normal operation of the airplane flight systems within the preceding 6 calendar months. B) emergency operation of all airplane flight systems within the preceding 12 calendar months. C) alternate operation of the airplane flight systems within the preceding 24 calendar months. 225 PLT400 FEX Which documents are required to be carried aboard each domestic air carrier flight conducted under CFR part 121? A) Load manifest and flight release. B) Dispatch release, load manifest, and flight plan. C) Maintenance release, weight and balance release, and flight plan. 226 PLT409 FEX Duty and rest period rules for domestic air carrier operations require that a flight crewmember A) not be assigned to any duty with the air carrier during a required rest period. B) be relieved of all duty for at least 48 hours during any 7 consecutive days. C) not be assigned to any duty for a period of at least 18 hours if the flight crewmember had been on duty aloft for 9 hours. 227 PLT438 FEX Above which cabin altitude must oxygen be provided for all persons during the entire flight? A) All crewmembers 10,000 feet; all passengers 12,000 feet. B) All crewmembers 12,000 feet; all passengers 15,000 feet. C) All crewmembers 10,000 feet; all passengers 15,000 feet. 228 PLT438 FEX How much supplemental oxygen must pressurized air carrier transport powered airplanes carry for each flight crewmember on flight deck duty when operating at flight altitudes above 10,000 feet? A) A minimum of 1 hours` supply. B) A minimum of 2 hours` supply. C) A minimum of 30 minutes` supply. 229 PLT439 FEX Which maintenance task may a flight engineer perform while operating under 14 CFR part 125? A) Landing light replacement if there is no certificated mechanic available. B) Remove, inspect, and replace a chip detector if the malfunction occurs in a remote area. C) Replenish hydraulic fluid in accordance with applicable regulations and the certificate holder`s manuals. 230 PLT410 FEX During what situation may an airplane requiring a flight engineer be operated under 14 CFR part 91? A) Test flight. B) Revenue cargo flight. C) Passenger flight with compensation. 231 PLT413 FEX What minimum amount of fuel should remain for turbine-powered airplanes after jettisoning with the main fuel control? Forty-five minutes at A) 75 percent maximum continuous power. B) maximum continuous power with the critical engine inoperative. C) maximum range speed after climbing from sea level to 10,000 feet. 232 PLT196 FEX When are ATIS broadcasts updated? A) Only when the ceiling or visibility changes by a reportable value. B) Every 30 minutes if weather conditions are below basic VFR, otherwise hourly. C) Upon receipt of any official weather, regardless of content change or reported values. 233 PLT493 FEX Which is an effect of ice, snow, or frost formation on an airplane? A) Increased stall speed. B) Increased pitch-down tendencies. C) Increased angle of attack for stalls. 234 PLT203 FEX Which is true concerning the troposphere? A) It extends to a uniform height at all latitudes. B) It is thicker over the Equator than over the poles. C) It is the dividing line between the stratosphere and the atmosphere. 235 PLT226 FEX The temperature and dewpoint spread is small and decreasing, and the temperature is +62 °F. Which type of weather is most likely to develop? A) Rain showers. B) Thunderstorms. C) Fog or low clouds. 236 PLT274 FEX An in-flight condition necessary for structural icing to form is A) visible moisture. B) stratiform clouds. C) cirrostratus clouds. 237 PLT173 FEX What atmospheric condition will decrease air density? A) Decreasing humidity. B) Decreasing pressure. C) Decreasing temperature. 238 PLT132 FEX What will be the approximate altimeter indication after failing to reset the local barometric pressure of 30.57 after descending from FL 250 to a field elevation of 650 feet? A) Sea level. B) 715 feet. C) 1,300 feet. 239 PLT313 FEX The CG of an airplane is normally located in the fuselage at a point expressed in A) inches from the forward CG limit. B) percent of mean aerodynamic chord aft of LEMAC. C) percentage of MAC aft of the leading edge of the wing. 240 PLT021 FEX (Refer to figure 40.) What is the loaded CG in percent of MAC under operating conditions No. 1? A) 28.9 percent. B) 30.5 percent. C) 32.9 percent. 241 PLT021 FEX (Refer to figure 52.) What is the maximum payload under operating conditions No. 1? A) 20,500 pounds. B) 21,500 pounds. C) 25,500 pounds. 242 PLT021 FEX What is the maximum payload under these conditions? A) 72,000 pounds. B) 80,000 pounds. C) 84,000 pounds. 243 PLT253 FEX A reason for using a crossfeed fuel system is to A) be able to purge any fuel tank. B) jettison fuel during emergencies. C) help maintain the aircraft`s center of gravity. 244 PLT021 FEX May 1,000 pounds of baggage be shifted from Station 30.0 to Station 120.0 without exceeding the aft CG limit? A) Yes, the CG would be located at Station 115.19. B) No, the new CG would be located at Station 118.41. C) Yes, the new CG would be located at Station 116.41. 245 PLT121 FEX What minimum weight of cargo must be shifted from the aft to the forward compartment to bring the CG within limits? A) 740 pounds. B) 1,032 pounds. C) 1,338 pounds. 246 PLT240 FEX If the landing gear on an airplane moves forward during retraction, the A) total moments will decrease. B) total moments will increase. C) total moments will remain the same. ac63-1.pdf FEXSampleExam.pdf fex.pdf